SlideShare una empresa de Scribd logo
1 de 107
Marc Imhotep Cray MD
2
Safety precautions: chemotherapy drugs are hazardous
For health professionals:
 Many chemotherapy drugs are considered hazardous to healthy
people. Reason why the nurses and doctors who give chemotherapy
will take precautions to avoid direct contact with the drugs while
administering them
 Chemotherapy drugs can be dangerous to others in these ways:
• They can cause abnormal changes in DNA. (They are mutagenic)
• They may be able to alter development of a fetus or embryo,
leading to birth defects (They are teratogenic)
• They may be able to cause another type of cancer. (They are
carcinogenic)
• Some may cause skin irritation or damage.
Nurses may wear special gloves, goggles, and gowns when preparing
and giving you chemotherapy
Pharmacists or nurses prepare the drugs in areas with special
ventilation systems to avoid spattering and/or inhaling the droplets
that can form while mixing
Source: ACS Chemotherapy Principles
Marc Imhotep Cray MD
How to Learn Chemotherapeutic Agent
3
Begin by learning drug classes (antimetabolites, alkylating agents,
antibiotic/anti-tumor agents, etc.) and their MOA
 Classify each individual drug according to these groups
 Learn toxicity of each individual ChemoTx agent . “Chemo man”
 Step 1 likes testing on toxicities of chemotherapeutic drugs.
 Briefly study drugs that can help neutralize toxic effects of
ChemoTx agents.
• Most important one to know is mesna, which can prevent
hemorrhagic cystitis in patients receiving cyclophosphamide.
Marc Imhotep Cray MD
How to learn Chemotherapeutic Agent (2)
4
 Learning clinical uses of chemotherapeutic agents should be last
on our list.
 You do not have to know which drugs are used for all types of
cancers, but there are a few on which you should focus.
• Know the drugs that are useful for testicular cancer (etoposide,
bleomycin, and cisplatin), choriocarcinoma (methotrexate and
vincristine/vinblastine), acute myelogenous leukemia (cytarabine), and
brain tumors (nitrosoureas)
• You should also know that 5-fluorouracil can be given topically for
actinic keratosis
Marc Imhotep Cray MD
Important Terms to know
5
 Cell cycle-nonspecific (CCNS) drug An anticancer agent that acts on tumor
stem cells when they are traversing cell cycle and when they are in resting
phase
 Cell cycle-specific (CCS) drug An anticancer agent that acts selectively on
tumor stem cells when they are traversing cell cycle and not when they are
in G0 phase
 Growth fraction The proportion of cells in a tumor population that are
actively dividing
 Myelosuppressant A drug that suppresses formation of mature blood cells
such as erythrocytes, leukocytes, and platelets. This effect is also known as
“bone marrow suppression”
 Oncogene A mutant form of a normal gene that is found in naturally
occurring tumors and which, when expressed in noncancerous cells, causes
them to behave like cancer cells
Marc Imhotep Cray MD
Useful Abbreviations
6
ADC Antibody-drug conjugate
Ara-C Cytarabine
DHFR Dihydrofolate reductase
FDA United States Food and Drug Administration
FH 4 Tetrahydrofolate
5-FU 5-Fluorouracil
Mesna Sodium 2-mercaptoethane sulfonate
6-MP 6-Mercaptopurine
MTX Methotrexate
6-TG 6-Thioguanine
Marc Imhotep Cray MD
CHEMOTHERAPY I: OVERVIEW
7
1. The importance of tumor cell heterogeneity and the development of
malignant cell resistance to chemotherapy as critical factors in determining
treatment outcome.
2. The process of antineoplastic drug development.
3. The criteria determining the response to antineoplastic agents.
4. The toxicity of cancer chemotherapy.
5. The principles of management of the patient with cancer.
6. The importance of tumor staging in the management of the patient with
cancer.
7. The principles of chemotherapy.
8. The rationale for the administration of adjuvant chemotherapy.
See: ACS Chemotherapy Principles Doc.
Core Concepts and Learning Objectives
Marc Imhotep Cray MD
Principles of Cancer
Chemotherapy
8
Marc Imhotep Cray MD
Major Approaches to Treatment of Cancers
9
Tumor cells are shown in red and nontumor cells in green. *In clinical use. Others are experimental
Brody Human Pharmacology: Mechanism-Based Therapeutics 6e. Elsevier, 2019. Pg. 1728.
Marc Imhotep Cray MD
Chemotherapy’s Main Clinical Usefulness
Chemotherapy is useful for disseminated cancers that cannot be
removed by surgery or as supplemental treatment after surgery or
radiation
 Using surgery or radiation to shrink tumor before ChemoTx ↑ number of
dividing cells which ↑ effectiveness of ChemoTx
 Most anticancer drugs affect cell division
• They act preferentially on rapidly proliferating cells
• Smaller tumors have a higher growth fraction
o Consequently, they are more susceptible to anticancer drugs
o Adjuvant ChemoTx is used w surgery or radiation to Tx undetectable metastases
when they are small and highly sensitive to anticancer drugs
• A greater proportion of nondividing cells will survive ChemoTx
compared to dividing cells
10
Marc Imhotep Cray MD
Chemotherapy’s Objective
11
Objective of ChemoTx in any given individual patient may be:
 Curative, to obtain complete remission (e.g., Hodgkin disease)
 Palliative, to alleviate symptoms but with little expectation of complete
remission (e.g., carcinoma of esophagus, with chemotherapy performed to
ease the dysphagia)
 Adjuvant, to improve chances for a cure or prolong period of disease-free
survival when no detectable cancer is present but subclinical numbers of
neoplastic cells are suspected (e.g., chemotherapy for breast cancer after
surgical resection of all known tumor)
 Debulking, to reduce tumor burden to allow for a more effective surgical
removal of the tumor
Marc Imhotep Cray MD
Therapeutic Overview
12
Cancers in Which Complete Remission to Chemotherapy Is
Common and Cures Are Achieved, Even in Advanced Disease*
 Acute lymphocytic leukemia (adults and children)
 Acute myelogenous leukemia
 Hodgkin disease (lymphoma)
 Non-Hodgkin lymphoma
 Choriocarcinoma
 Testicular cancer
 Burkitt lymphoma
 Ewing sarcoma
 Wilms tumor
 Small cell lung cancer
 Ovarian cancer
 Hairy cell leukemia
*Depending on tumor type, complete
remission may result in cure
Marc Imhotep Cray MD
Therapeutic Overview (2)
13
Cancers in Which Objective Responses Are Achieved, but
Chemotherapy Often Does Not Have Curative Potential in
Advanced Disease
 Multiple myeloma
 Breast cancer
 Head and neck cancer
 Colorectal carcinomas
 Chronic lymphocytic leukemia
 Chronic myelogenous leukemia
 Transitional cell carcinoma of bladder
 Gastric adenocarcinomas
 Cervical carcinomas
 Melanoma
 Neuroblastoma
 Non–small cell lung cancer
Marc Imhotep Cray MD
Therapeutic Overview (3)
14
Cancers in Which Only Occasional Objective Responses to
Chemotherapy Are Achieved
 Renal tumor
 Pancreatic carcinomas
 Hepatocellular carcinoma
 Prostate carcinomas (hormone-unresponsive)
15
Basic Mechanisms by Which Antineoplastic Drugs Selectively
Kill Tumor Cells
 E represents enzymes, some of which
are inhibited by these drugs
 Key mechanisms of action for many
of these drugs are:
• Inhibition of DNA or RNA
synthesis,
• Production of miscoded nucleic
acids, and
• Formation of modified proteins
Brody Human Pharmacology: Mechanism-Based Therapeutics 6e. Elsevier, 2019. Pg. 1748.
16
Cell Cycle Kinetics
 Cancer cell population kinetics and cancer cell
cycle are important determinants of actions
and clinical uses of anticancer drugs
• Some anticancer drugs exert their actions
on cells undergoing cycling (cell cycle-
specific [CCS] drugs), and
• others (cell cycle-nonspecific [CCNS]
drugs) kill tumor cells in both cycling and
resting phases of cell cycle (although
cycling cell are more sensitive)
 CCS drugs are usually most effective when cells
are in a specific phase of cell cycle
 Both types of drugs are particularly effective
when a large proportion of tumor cells are
proliferating (ie, when growth fraction is high)
Katzung BG, editor: Basic & Clinical Pharmacology, 12th ed.
McGraw-Hill, 2012: Fig. 54–2
Marc Imhotep Cray MD
Cell Cycle Specific Anticancer Drugs
17
 They are only effective against replicating cells, particularly malignancies w
a high growth fraction
 The cell cycle phases include:
i. G1, phase after mitosis Some G1 cells can move into a resting,
non-dividing state, G0
ii. S, DNA synthesis phase
iii. G2, phase before mitosis
iv. M, mitotic phase
 Cell cycle phase specific drug classes include antimetabolites,
bleomycin peptide antibiotics, vinca alkaloids (microtubule inhibitors),
and etoposide
i. Folic acid analog methotrexate (MTX) kills in S-phase (DNA synthesis phase)
ii. Vincristine and vinblastine kill in M-phase (mitotic phase)
Marc Imhotep Cray MD
Cell Cycle Nonspecific Anticancer Drugs
18
Other drugs are cell cycle phase nonspecific
 They are effective at killing nondividing cells as well as
dividing cells
 Cell cycle phase nonspecific drug classes include alkylating
agents, most anticancer antibiotics, cisplatin, and
nitrosoureas
Marc Imhotep Cray MD
The four phases of the cell cycle. G1 – the initial growth phase. S – the phase
in which DNA is synthesized. G2 – the second growth phase in preparation for
cell division. M – mitosis; where the cell divides to produce two daughter cells
that continue the cell cycle.
19
Marc Imhotep Cray MD
Cancer therapy—cell cycle
20
First AID for the USMLE Step 1 2020, Pg. 438.
Marc Imhotep Cray MD
Cancer therapy—targets
21
First AID for the USMLE Step 1 2020, Pg. 438.
Marc Imhotep Cray MD 22
 Proportion of cancer cells that are actively proliferating will predict response
to ChemoTx
 Cells not actively proliferating are generally unresponsive to ChemoTx
 As cancers increase in size, more cancer cells exit replicative pool
 During course of ChemoTx , cells in replication are selectively killed, leaving
behind cells out of pool that can continue on
• If one were to completely surgically excise mass, then that would solve this issue, but
even if mass cannot be removed, then at least one can debulk it, or use radiation
therapy to kill more cells
 ChemoTx protocols can be designed to include more agents, thereby
reducing cancer cell resistance, or by giving cycles of ChemoTx that get
more cancer cells into replicative pool
CANCER CELLS ARE KILLED IN A FIRST-ORDER MANNER  a constant
percentage (or proportion) of cancer cells are killed w each course of therapy
 b/c of this log kill, additional rounds of chemotherapy are necessary in order to
completely eradicate the tumor
Marc Imhotep Cray MD
The Log-Kill Hypothesis
23
Cytotoxic drugs act with first-order kinetics in a murine model of
leukemia
 In this model system a given dose kills a constant proportion of a cell
population rather than a constant number of cells
 The log-kill hypothesis proposes that magnitude of tumor cell kill by
anticancer drugs is a logarithmic function
For example, a 3-log-kill dose of an effective drug reduces a cancer cell
population of 1012 cells to 109 (a total kill of 999 × 109 cells
the same dose would reduce a starting population of 106 cells to 103
cells (a kill of 999 × 103 cells)
• In both cases, dose reduces numbers of cells by 3 orders of magnitude, or “3 logs”
• A key principle that stems from this finding and that is applicable to hematologic
malignancies is an inverse relationship betw. tumor cell number and curability
24
Cancer Log-kill hypothesis
Relationship, based on the log-kill hypothesis, of
tumor cell number to 3 approaches to drug
treatment and to no treatment (dashed line).
 In the protocol diagrammed at top, infrequent
treatment (indicated by arrows) prolongs survival
but with recurrence of symptoms betw. Tx and
eventual death.
 With regimen diagrammed in middle section that
is more intensive and begun earlier, cure results
after many cycles of therapy.
 In treatment diagrammed near bottom of graph,
early surgery removes much of tumor burden,
and intensive adjuvant chemotherapy has been
used long enough to produce a cure.
Katzung BG, editor: Basic & Clinical Pharmacology, 12th ed.
McGraw-Hill, 2012: Fig. 54–1
Marc Imhotep Cray MD
Standard toxicities anticancer drugs
25
1. Myelosuppression is common because bone marrow is a
rapidly proliferating tissue
a. This is usually the dose-limiting side effect
b. The leukopenia is greater than the thrombocytopenia, which is
greater than the anemia
c. Drugs for which bone marrow depression is not the dose-limiting
toxicity include:
i. Hormones
ii. Vincristine
iii. Bleomycin
iv. Asparaginase
v. Cisplatin
vi. Monoclonal antibodies (MAbs)
Marc Imhotep Cray MD
Standard toxicities anticancer drugs (2)
26
2. Other rapidly proliferating cells that are affected include:
a. GI epithelium
b. Germinal epithelium
c. Hair follicles
Marc Imhotep Cray MD
Standard toxicities anticancer drugs (3)
27
3. Nausea and vomiting are common side effects that can be
managed with antiemetics, including:
a. Phenothiazines such as prochlorperazine (Compazine)
b. Cannabinoids such as dronabinol (Marinol)
c. Dopamine receptor antagonists such as metoclopramide (Reglan)
d. Ondansetron (Zofran), a 5-HT3 antagonist
e. Glucocorticoids, such as dexamethasone
f. Antihistamines, such as diphenhydramine (Benadryl)
g. Benzodiazepines, such as lorazepam (Ativan)
Marc Imhotep Cray MD
Standard toxicities anticancer drugs (4)
28
4. Tissue necrosis may occur at site of injection
5. Some anticancer drugs have unique organ toxicities
a. Anthracyclines (doxorubicin, daunorubicin, idarubicin, epirubicin and
mitoxantrone) are cardiotoxic
b. Bleomycin induces pulmonary fibrosis
c. Vinca alkaloids (vincristine, vinblastine and vinorelbine); platinum
compounds (cisplatin, carboplatin and oxaliplatin); and taxanes (paclitaxel and
docetaxel) are neurotoxic
d. Cisplatin, carboplatin, and methotrexate are nephrotoxic
Anthracyclines (eg, Doxorubicin, Daunorubicin); prevent w Dexrazoxane
Marc Imhotep Cray MD
Minimizing ChemoTx Toxicities
29
6. Adverse side effects can be minimized by a variety of
techniques including:
a. Local perfusion of tumors
b. Removing marrow pre-treatment and reimplanting it afterward
c. Diuresis to prevent bladder toxicity
d. Administering leucovorin (folinic acid) for megaloblastic anemia and
prevention of MTX toxicity
e. Urine alkalinization for MTX excretion
f. Administering G-CSF (filgrastim) for neutropenia
g. Administering allopurinol (Zyloprim) or rasburicase (Elitek) to treat
hyperuricemia associated with tumor lysis syndrome, especially in leukemia
and lymphoma patients
30
Common Cancer Chemotherapy Toxicities
Drug Side Effect / Toxicity Mechanism
Cyclophosphamide Hemorrhagic cystitis acrolein metabolite damages bladder epithelium (Rx: Mesna)
5-FU
Diarrhea,
photosensitivity
damage of rapidly dividing GI epithelial cells interferes with fluid absorption from GI tract,
resulting in watery bowel movements; 5-FU is “phototoxic” when exposed to sunlight
Bleomycin Pulmonary Toxicity
may be related to a combination of the production of toxic oxygen-free radicals & DNA
damage in the lung
Vincas
Neuropathy,
constipation
Neuropathy may result from effects on neuronal microtubular assembly; constipation may in
turn result from GI neuropathy
Taxanes Neuropathy believed to result from disruption of neuronal microtubules
Daunorubicin
(Adriamycin)
Cardiac toxicity
evidence suggests that reactive oxygen species, generated by the interaction of doxorubicin
with iron, damage cardiac myocytes, causing cellular damage & loss of muscle fibers, which
can result in CHF
Cisplatin Emesis, renal toxicity
cisplatin is the most highly emetogenic agent; a proposed mechanism is that free radicals
produced by cisplatin act on enterochromaffin cells to cause release of serotonin (5-HT); 5-
HT3 receptor antagonists (e.g. ondansetron) are used to treat cisplatin induced emesis; renal
toxicity may result from the ability of proximal tubular kidney cells to metabolize cisplatin to a
Nephrotoxin, resulting in kidney damage
Tamoxifen
Uterine cancer,
thrombosis
Tamoxifen is a SERM and the increase in uterine (endometrial) cancer risk is related to
tamoxifen's effect to behave as an estrogen partial agonist in the endometrium; the increase
in deep vein thrombosis appears to result from an effect of tamoxifen to increase platelet
aggregation by a poorly understood mechanism
Alkylating agents Secondary leukemias DNA damage & resulting mutations caused by alkylating agents
Marc Imhotep Cray MD
Mesna (Mesnex)
31
Chemotherapy adjuvant
 Mesna is used therapeutically to reduce incidence of hemorrhagic cystitis
and hematuria when a patient receives ifosfamide or cyclophosphamide for
cancer chemotherapy
• These two anticancer agents, in vivo, may be converted to urotoxic
metabolites, such as acrolein
 MOA Mesna assists to detoxify these metabolites by reaction of its
sulfhydryl group with α, β-unsaturated carbonyl containing compounds such
as acrolein
32
Key Chemotoxicities “CHEMO MAN”
First AID for the USMLE Step 1 2020, Pg. 444.
Marc Imhotep Cray MD
Other Problems with Chemotherapy
33
1. Immunocompromised patients usually have poorer responses
to anticancer treatment
2. Centers of large tumors and CNS can serve as pharmacologic
sanctuaries
 Cancer cells are relatively protected from cytolytic effects of systemic
chemotherapy
3. Some ChemoTx agents, particularly alkylating agents, can cause
new, treatment-induced cancers up to several years after treatment
 teratogenicity can also occur, again especially with alkylating
agents
Marc Imhotep Cray MD
Other Problems with ChemoTx (2)
34
4. Resistance can develop
a. Some cancers are inherently resistant to certain agents; other cancers
can develop resistance by mutation, especially after long-term
administration of low doses of drug
b. Resistance is minimized by short-term, intensive, intermittent Tx w
combinations of drugs
c. Multidrug resistance occurs due to stepwise selection for permeability
glycoprotein (P glycoprotein)
i. P glycoprotein actively pumps drugs out of the cell
ii. b/c P glycoprotein is a multidrug efflux pump, its activity provides cross-resistance
for several structurally unrelated drug classes
iii. Some organs naturally express high levels of P glycoprotein, including kidneys,
intestines, liver, and pancreas Cells of these organs are therefore more resistant
to chemotherapy
Marc Imhotep Cray MD
The Multidrug Resistance (MDR) Gene:
Drug Specificity and Tissue Distribution
35
Drugs Affected by MDR Drugs Not Affected by MDR
Adriamycin Methotrexate
Daunomycin 6-Thioguanine
Dactinomycin Cytarabine
Plicamycin Cyclophosphamide
Etoposide BCNU
Vinblastine Bleomycin
Vincristine Cisplatin
Tissues with High MDR Expression Tissues with Low MDR Expression
Colon Bone marrow
Liver Breast
Pancreas Ovary
Kidney Skin
Adrenal Central nervous system
Marc Imhotep Cray MD
Combination Chemotherapy
36
Combination Therapy is common and is often more effective
against a wider variety of cell lines
1. Each drug in combination should be active against tumor to provide
maximum cell killing within range of tolerance
2. Drugs should have different mechanisms of action to kill maximum
number of cells in heterogeneous tumors
3. Drugs should have different toxicities so that they can all be given at
full strength and emergence of resistance can be delayed
4. Drugs are usually admin. in Tx cycles and time must be allowed for host
tissue recovery betw. cycles
Marc Imhotep Cray MD
Examples of Common Combination Drug Regimens
37
Terminology
Cancer Drugs
MOPP Hodgkin lymphoma
Mechlorethamine, vincristine, procarbazine,
prednisone
ABVD Hodgkin lymphoma Doxorubicin, bleomycin, vinblastine, dacarbazine
BEP Germ cell cancers Bleomycin, etoposide, cisplatin
CMF Breast cancer Cyclophosphamide, methotrexate, 5-fluorouracil
CAF Breast cancer Cyclophosphamide, doxorubicin, 5-fluorouracil
CHOP
CHOP-R
Non-Hodgkin lymphoma
Cyclophosphamide, doxorubicin, vincristine,
prednisone Agents above + rituximab
Hyper-CVAD
Non-Hodgkin
lymphoma
Hyperfractionated cyclophosphamide, vincristine,
adriamycin, dexamethasone
FOLFIRI
Cetuximab
Colorectal cancer Leucovorin, 5-fluorouracil, irinotecan, cetuximab
PVC Anaplastic oligodendrogliomas Procarbazine, vincristine, lomustine
R-EPOCH B-cell non-Hodgkin lymphoma
Rituximab, etoposide, prednisone, vincristine,
cyclophosphamide, doxorubicin
XELIRI Colorectal, esophageal, gastric cancer Capecitabine, irinotecan
Brody Human Pharmacology: Mechanism-Based Therapeutics 6e. Elsevier, 2019. Pg. 1736.
Marc Imhotep Cray MD
Targeted therapies & Immunopharmacology
38
Targeted therapies are becoming increasingly important in the treatment of
cancer
 Examples include bevacizumab, which targets vascular endothelial
growth factor;
 I131 tositumomab, rituximab, and Y-90-ibritumomab tiuxetan, which
target CD20 and are used for treatment of chemotherapy-refractory non-
Hodgkin's lymphoma; and
 Gefitinib and antibody cetuximab, which target epidermal growth factor
receptor pathway
 Immunotherapy includes biological response modifiers, which stimulate
human immune system to destroy tumor cells
 The α and β human interferons are examples of efficacious agents for
hairy cell leukemia and certain skin cancers
 Other compounds include tumor necrosis factor, human growth factors,
and monoclonal antibodies
Marc Imhotep Cray MD
Targeted therapies & Immunopharmacology (2)
39
Notably important immuno-oncology targets are programmed death
ligand 1 (PD-L1) and its cognate receptor, PD-1, which is found on
activated T cells, B cells, and myeloid cells
 PD-L1 up regulation allows melanoma, lung, renal, and gastric cancers
to evade the host immune system PD-1 checkpoint inhibitors, such
as nivolumab and pembrolizumab, have been approved
Another example of effective immuno-oncology therapy is blocking
antibody against cytotoxic T-lymphocyte–associated protein 4 (CTLA-4)
Ipilimumab, which down regulates cytotoxic T-lymphocytes and is used for
treatment of melanoma
Learn more about Targeted Therapies & CA Immunopharmacology in
Cancer Chemotherapeutic Drugs Notes_Pg. 28-40
Marc Imhotep Cray MD 40
Chemotherapeutic Agents
A Case-based Discussion
Companion Notes:
Cancer Chemotherapy Notes.pdf
Cancer Chemotherapeutic Agents Rapid Review Q&A.pdf
Marc Imhotep Cray MD 41
Lange Pharmacology Flashcards, Fourth Edition, M-H 2018. Pg.264.
Marc Imhotep Cray MD
Classification of Anti-Neoplastic Agents by Site of Action
42
Marc Imhotep Cray MD
Classification of Anti-Neoplastic Agents by Site of Action
43
(See Reproductive)
Marc Imhotep Cray MD
CHEMOTHERAPY II: ALKYLATING AGENTS
44
1. The characteristic indications for the alkylating
agents and plant alkaloids commonly used in
chemotherapy.
2 .The mechanism of action and mechanisms of
resistance for the alkylating agents and plant alkaloids
commonly used in chemotherapy.
3. The principal adverse effects of the commonly used
alkylating agents and plant alkaloids.
Relevant Drugs:
Cyclophosphamide
Ifosfamide
Cisplatin
Carboplatin
Oxaliplatin
Busulfan
Nitrosureas
Procarbazine
Core Concepts and Learning Objectives
Marc Imhotep Cray MD
Case#1
45
A 72-year-old woman presents to the emergency room complaining
of the onset of pain and bleeding with urination over the past 24
hours. She denies any fever, flank pain, or history of kidney stones.
Her past medical history is significant for breast cancer, for which
she is currently receiving chemotherapy. Urinalysis is significant for
copious amounts of blood and red blood cells. You admit the
patient to the hospital for a urology workup and you begin to
wonder if her symptoms might be related to her chemotherapeutic
regimen.
What is the Medication?
Marc Imhotep Cray MD
Cyclophosphamide
46
Similar Drugs
Ifosfamide
Mechanism of Action
 Cyclophosphamide is metabolized to a hydroxylated intermediate
compound by cytochrome P-450 system in liver This hydroxylated
intermediate then acts as an alkylating agent and cross-links DNA,
thereby decreasing DNA and RNA synthesis
 drug has also been implicated in the suppression of B- and T-cell function
Clinical Uses
 Used in treatment of a variety of solid and hematologic malignancies
(e.g., breast, ovarian, non-Hodgkin lymphoma)
 Immunosuppressive agent used to treat rheumatoid arthritis, SLE,
Wegener granulomatosis, and nephrotic syndrome
Marc Imhotep Cray MD
Cyclophosphamide (2)
47
Side Effects/AEs
Hemorrhagic cystitis (caused by accumulation of the metabolite, acrolein, in
urine; incidence can be greatly reduced by giving patient copious fluids as well
as MESNA [sodium 2-mercaptoethane sulfonate]); bone marrow suppression;
GI upset; alopecia; infertility
Other Mechlorethamine is another alkylating agent that is used as part of
MOPP protocol (mechlorethamine, vincristine, procarbazine, and prednisone)
in Tx of Hodgkin’s lymphoma.
Procarbazine is another alkylating agent that is also used as part of MOPP
protocol in treatment of Hodgkin’s disease as well as some brain tumors.
 A metabolite of procarbazine inhibits monoamine oxidase, and thus
patients taking this drug should not ingest foods containing tyramine (wine,
smoked meat, cheese) due to risk of norepinephrine buildup causing a
hypertensive crisis
Marc Imhotep Cray MD
Case #2
48
A 51-year-old man presents to your oncology clinic for evaluation of his CML.
He will be undergoing bone marrow transplantation in the coming weeks,
and he will need to have his marrow ablated prior to his transplantation. You
inform him that his bone marrow will be ablated with a combination of
medications, including an alkylating agent that acts to damage DNA via the
cross-linking of DNA strands. You explain to him that this medication has
many side effects, including adrenal insufficiency, seizures, and pulmonary
fibrosis. The patient realizes that there may be serious side effects, but is
more than willing to undergo the treatment to prepare for potentially life-
saving bone marrow transplantation.
What is the Medication?
Marc Imhotep Cray MD
Busulfan
49
Mechanism of Action
Busulfan acts as an alkylating agent that acts to damage DNA via
cross-linking of DNA strands
Clinical Uses
 Treatment of CML
 Also used in combination with other drugs to ablate bone marrow
before bone marrow transplantation
Side Effects/AEs
Pulmonary fibrosis; bone marrow suppression; adrenal
insufficiency with associated hyperpigmentation; seizures
Black Box Warning: Busulfan is a potent cytotoxic drug that causes PROFOUND MYELOSUPPRESSION
at the recommended dosage. It should be administered under the supervision of a qualified physician
who is experienced in allogeneic hematopoietic stem cell transplantation, the use of cancer
chemotherapeutic drugs and the management of patients with severe pancytopenia.
Marc Imhotep Cray MD
Case#3
50
A 42-year-old man presents to your neurology clinic complaining of severe
headaches that have been present almost every day for the past month. The
pain is usually lateralized to the right side and is often present upon waking.
Over the past few days, he has also experienced several episodes of projectile
vomiting. Physical examination is significant for the presence of papilledema on
funduscopic examination. You become concerned and send the patient for an
MRI of the brain. The imaging study reveals a mass in the right temporal lobe.
Characteristics of the mass on imaging are consistent with a likely malignancy.
You refer the patient to a neurosurgeon as well as to an oncologist. You believe
that he will need chemotherapy and radiation to treat his condition and you
suspect that his chemotherapy regimen will likely include a class of drugs that
can cross the blood-brain barrier and then cross-link DNA strands to inhibit
DNA synthesis.
What is the Medication?
Marc Imhotep Cray MD
Nitrosoureas
51
Similar Drugs
Nitrosoureas include carmustine, lomustine, semustine, and streptozocin
Mechanism of Action
Drugs in this class are alkylating agents, which act to cross-link DNA strands,
and thereby result in the inhibition of DNA and RNA synthesis
Clinical Uses
 Chemotherapeutic agents used in the treatment of brain tumors (e.g.,
glioblastoma multiforme), multiple myeloma, and lymphomas.
 Streptozocin is also used in the treatment of insulinomas
Side Effects/AEs
CNS toxicity; bone marrow suppression; nephrotoxicity; pulmonary fibrosis.
Marc Imhotep Cray MD
Nitrosoureas (2)
52
Black Box Warnings:
BONE MARROW SUPPRESSION - notably thrombocytopenia & leukopenia. CARMUSTINE
PULMONARY TOXICITY is dose related. Risk increases with cumulative doses > 1400
mg/m2, history of lung disease & duration of therapy. Delayed cases of pulmonary fibrosis
have been reported 15 years after administration.
Of Note: Nitrosoureas are one of the few chemotherapeutic
agents that are highly lipid-soluble and cross the blood-brain
barrier, and thus are useful in treating CNS malignancies
Marc Imhotep Cray MD
Case#4
53
A 76-year-old man presents to your clinic complaining of a ringing in both
of his ears that has been present over the past month. He also tells you that
he seems to be having more trouble hearing during this period as well. His
past medical history is significant for bladder cancer, for which he is
currently being treated with chemotherapy. Physical examination is
remarkable for gross sensorineural hearing loss bilaterally. You suspect that
his hearing loss is related to one of his medications and you decide to call his
oncologist to see if there is an alternative chemotherapeutic agent that can
be used to treat his malignancy, but will not cause acoustic nerve damage.
What is the Medication?
Marc Imhotep Cray MD
Cisplatin (cis–diamminedichloroplatinum)
54
Similar Drugs
Carboplatin
Mechanism of Action
Cisplatin acts as an alkylating agent, thereby inhibiting DNA and RNA synthesis
by binding DNA strands and leading to formation of cross-links between strands
Clinical Uses
Chemotherapeutic agent used in treatment of many genitourinary tumors
(testicular, ovarian, bladder), other solid tumors (small cell lung cancer,
stomach, esophageal) and lymphomas
Side Effects/AEs
Nephrotoxicity; ototoxicity; neurotoxicity; bone marrow suppression
Marc Imhotep Cray MD
Cisplatin(2)
55
Black Box Warnings: NEPHROTOXICITY, MYLEOSUPPRESSION & OTOTOXICITY
1.NEPHROTOXICITY is cumulative, dose related and severe. It has been noted in 28-36% of
patients treated with a single dose of 50 mg/m2. It is first noted during the 2nd week after a
dose & is manifested by elevations in BUN, creatinine & serum uric acid. Renal function must
return to normal before another dose of cisplatin can be given.
2.MYLEOSUPPRESSION: dose related and severe. Occurs in 25-30% of patients. Nadirs in
circulating platelets & leukocytes occur within 18-23 days in most patients, with recovering
by day 39.
3.Nausea & vomiting - dose related and severe. Seen in almost all patients & begins within
1-4 hrs after treatment & may last for 24 hrs or more (may persist up to 1 week).
4.OTOTOXICITY is significant (observed in up to 31% of patients treated with a single dose of
50 mg/m2) - may be more pronounced in children
5.Anaphylactic like reactions have been reported (epinephrine, corticosteroids &
antihistamines have been effective in alleviating symptoms).
6.Avoid overdose or confusion with carboplatin
Marc Imhotep Cray MD
Cisplatin(3)
56
Important Notes:
Cisplatin-induced nephrotoxicity is dose-dependent and can be
mitigated with use of amifostine acts as a scavenger of free
radicals in tissues treated w cisplatin, as well as aggressive IV fluids
to induce diuresis
Mitomycin is another alkylating agent that damages DNA via cross-
linking of DNA strands
 It is mostly used in combination with other agents to treat
certain solid malignancies (e.g., breast, gastric, esophageal,
pancreas, bladder)
Marc Imhotep Cray MD
CHEMOTHERAPY III: ANTI-METABOLITES
57
1. The characteristic indications for the use on
antimetabolites commonly used in the treatment of cancer.
2. The mechanism of action of the antimetabolites
commonly used in the treatment of cancer.
3. The mechanism of resistance to antimetabolites
4. The major adverse effects associated with
antimetabolites commonly used in the treatment of cancer.
Relevant Drugs
Methotrexate
Pemetrexed
Cytarabine
Gemcitabine
5-Fluorouracil
Capecitabine
6-Mercaptopurine
6-Thioguanine
Core Concepts and Learning Objectives
Marc Imhotep Cray MD
Case#5
58
A 42-year-old woman presents to your rheumatology clinic for follow-up of
her rheumatoid arthritis. She is currently being treated with nonsteroidal
anti-inflammatory drugs (NSAIDs) and steroids, but she states that her
symptoms are still severe. Physical examination demonstrates swelling of the
proximal interphalangeal and metacarpophalangeal joints of both hands,
ulnar deviation of the fingers, and subcutaneous nodules over both of her
elbows. You decide to start the patient on a medication that acts to inhibit
dihydrofolate reductase and will thereby decrease her body’s supply of
activated folic acid.
What is the Medication?
Marc Imhotep Cray MD
Methotrexate
59
Mechanism of Action
 MTX inhibits dihydrofolate reductase, enzyme that transforms folic acid to
active form, FH4 FH4 is a necessary precursor for formation of thymidylate
 Without FH4, thymidylate supplies are diminished, thereby resulting in
decreased DNA synthesis
 MTX is a S phase specific ChemoTx agent
Clinical Uses
 Chemotherapeutic agent used to treat a variety of malignancies (e.g., ALL, lymphomas,
breast cancer, head and neck cancer, osteosarcoma, choriocarcinoma).
 Immunosuppressant used to treat many autoimmune disorders (e.g., RA, Crohn’s disease,
scleroderma, psoriasis)
 Abortifacient when administered with a prostaglandin (used to treat missed abortions or
small ectopic pregnancies)
Side Effects/AEs
Bone marrow suppression; nephrotoxicity (may crystallize in renal tubules);
hepatotoxicity (steatosis); teratogen; pulmonary toxicity; mucositis; seizures;
GI upset; alopecia
Marc Imhotep Cray MD
MTX (2)
60
Important Notes:
 Leucovorin (folinic acid) is often given in conjunction w MTX in order to
minimize bone marrow suppression, which occurs b/c of lack of folate
coenzymes
 In the cell, leucovorin is converted to N5N10- methylene-FH4, which is a
downstream product of reaction catalyzed by dihydrofolate reductase
• Therefore, leucovorin acts to bypass the inhibited enzyme and provides an adequate
supply of activated folate
 MTX has same molecular target as antibiotic, trimethoprim, and anti-parasitic
drug, pyrithiamine, but MTX work to target different isoforms of
dihydrofolate reductase
Marc Imhotep Cray MD 61
Black Box Warnings (abbreviated version):
1.Because of the possibility of serious toxic (and potentially fatal) reactions, methotrexate should be
used only in life threatening neoplastic diseases or severe recalcitrant psoriasis or rheumatoid
arthritis which is not responsive to other forms of therapy.
2.Include the use of antimetabolite (Leucovorin) therapy
3.Patients should be closely monitored for bone marrow, liver, kidney & lung toxicity.
4.Methotrexate has been reported to cause fetal death & congenital abnormalities. Not
recommended for women of child bearing potential unless the benefits outweigh the risks.
5.Unexpected bone marrow suppression, aplastic anemia & GI toxicity have been reported with high
doses of methotrexate & concurrent use of NSAIDs.
6.Methotrexate causes liver toxicity, fibrosis & cirrhosis, but generally only after prolonged use.
7.Malignant lymphomas, which may regress following withdrawal of methotrexate may occur with
low dose treatment.
8.Diarrhea & ulcerative stomatitis require interruption of therapy, otherwise hemorrhagic enteritis &
death from intestinal perforation may occur.
9.Similar to other cytotoxic drugs, methotrexate cay cause a “tumor lysis syndrome” in patients with
rapidly growing tumors. Appropriate measures should be taken should this occur.
10.Severe & occasionally fatal skin reactions have been reported.
11.Potentially fatal opportunistic infections (e.g. Pneumocystis jiroveci pneumonia) may occur.
12.Methotrexate combined with radiotherapy may increase the risk for necrosis of soft tissues &
osteonecrosis.
Marc Imhotep Cray MD
Case#6
62
A 10-year-old girl presents to your oncology clinic for follow-up of her acute
lymphoblastic leukemia, for which she is currently receiving chemotherapy. She
was recently discharged from the hospital 2 days prior to her appointment at
your clinic. Her mother tells you that the patient developed an episode of
acute gout while in the hospital, after which she was placed on allopurinol to
prevent future gouty attacks. You immediately decide to alter the doses of the
patient’s chemotherapeutic regimen, as allopurinol indirectly inhibits the
metabolism of one of the drugs in the patient’s chemotherapeutic regimen and
can thus result in increased serum levels of this drug.
What is the Medication?
Marc Imhotep Cray MD
6-Mercaptopurine
63
Similar Drugs
Azathioprine
Mechanism of Action
6-MP is converted into thio-IMP (6-Thioinosine 5'-monophosphate) by HGPRT
(Hypoxanthine-guanine phosphoribosyltransferase), an enzyme involved in the
salvage pathway. Thio-IMP acts as a purine analogue, which inhibits purine
synthesis through feedback inhibition of several enzymes involved in de novo
purine synthesis. Thio-IMP is also converted to thio-GTP, which is incorporated
into DNA and RNA, leading to strand instability
Clinical Uses
Chemotherapeutic agent used primarily in treatment of leukemias (especially
acute lymphoblastic leukemia) as well as non-Hodgkin lymphoma
Immunosuppressant used to treat inflammatory bowel disease and psoriasis and
prevent allograft rejection
Side Effects/AEs
Bone marrow suppression; GI upset; increased susceptibility to infection
Marc Imhotep Cray MD
6-MP
64
Improtant Notes:
DDI: 6-MP is metabolized by xanthine oxidase, an enzyme involved in purine
degradation Allopurinol, a drug used to treat hyperuricemia and gout, is a
potent inhibitor of xanthine oxidase
 When allopurinol and 6-MP are given simultaneously levels of serum 6-
MP become ↑ b/c of ↓ metabolism of 6-MP by xanthine oxidase
Azathioprine a structural analogue of 6-MP & isconverted to 6-MP within cell
 It is primarily used as an immunosuppressant in autoimmune diseases such
as SLE, IBD, RA, ITP, multiple sclerosis, and allograft rejection
 Side effects are similar to those caused by 6-MP, and include bone marrow
toxicity as well as GI upset
 Both azathioprine and 6-MP are S-phase specific chemotherapeutic agents
Marc Imhotep Cray MD
Case #7
65
A 58-year-old man presents to your office for a follow-up visit. He was
recently diagnosed with colon cancer and underwent colonic resection. He is
currently receiving chemotherapy as an adjuvant treatment. At this visit, he
tells you that he has noticed increased sensitivity of his eyes to light as well
as painful ulcers on his lips and in his mouth. After examining his eyes and
oral pharynx, you tell him that you suspect that his photosensitivity and
mucosal ulcers are likely a side effect of one of the chemotherapeutic drugs
being used to treat his colon cancer.
What is the Medication?
Marc Imhotep Cray MD
5-Fluorouracil
66
Mechanism of Action
5-FU is converted into 5-FdUMP, which then acts to inhibit thymidylate
synthase, enzyme responsible for synthesizing thymine nucleotides Inhibition
of thymidylate synthase leads to a disruption of nucleotide synthesis b/c of a
lack of thymidine; 5-FU can also be incorporated into RNA, leading to
dysfunctional RNA processing
Clinical Uses
Chemotherapeutic agent used to treat a variety of adenocarcinomas (gastric,
pancreatic, colon, breast, ovarian) and for basal cell carcinoma
Side Effects/AEs
Bone marrow suppression; photosensitivity; anorexia; oral ulcerations.
Of Note 5-FU is a cell cycle-specific drug, acting during S phase of cell cycle
Marc Imhotep Cray MD
Case#8
67
A 36-year-old woman presents to your oncology clinic for follow-up of her
acute myelogenous leukemia. She has been receiving several chemotherapeutic
agents for the treatment of her disease. Her most recent treatment involved a
cell cycle-specific drug that acts by competitively inhibiting DNA polymerase.
She tells you that lately she has been feeling extremely tired and that she had a
nosebleed last week that took a long time to stop bleeding. On physical
examination, you note that she is a pale, thin woman with a 3/6 systolic flow
murmur at the left upper sternal border and a diffuse petechial rash over her
limbs. Concerned, you send her to the emergency room for blood tests, which
you suspect will reveal pancytopenia that is most likely caused by her recent
chemotherapy treatment.
What is the Medication?
Marc Imhotep Cray MD
Cytarabine (cytosine arabinoside)
68
Mechanism of Action
In the cell, cytarabine is converted to araCTP (arabinofuranosylcytosine
triphosphate), which acts to competitively inhibit DNA polymerase and thus
impair DNA synthesis. araCTP can also be incorporated into DNA, leading to
termination of DNA strand elongation during DNA synthesis
 Cytarabine is a cell cycle-specific agent, acting during S phase of cell cycle
Clinical Uses
Chemotherapeutic agent for the treatment of AML as well as lymphomas
Side Effects/AEs
Bone marrow suppression with resulting pancytopenia;
alopecia; GI upset; ataxia
Black Box Warnings: The main toxic effect of cytarabine injection
is BONE MARROW SUPPRESSION with leukopenia, thrombocytopenia,
and anemia.
Marc Imhotep Cray MD
Cladrabine
69
Cladrabine acts by interfering with DNA synthesis as well; After becoming
activated through phosphorylation, cladrabine incorporates into growing
DNA strands and leads to DNA strand breakage
 Because activated form of drug also inhibits ribonucleotide
reductase, it also induces a depletion of adenosine triphosphate (ATP)
 Cladrabine is a cell cycle-non-specific agent, and thus can induce cell
death in resting as well as proliferating cancer cells
USE It is used primarily for treatment of hairy cell leukemia
Side effects/AEs include bone marrow suppression, neurotoxicity, and
nephrotoxicity.
Marc Imhotep Cray MD
CHEMOTHERAPY V: NATURAL PRODUCTS:
PLANT ALKALOIDS
70
Vinca alkaloids: Vincristine, Vinblastine and Vinorelbine.
Taxanes: Paclitaxel and Docetaxel
Podophyllotoxins: Etoposide and Tenisopide
Camptothecan analogs: Irinotecan and Topotecan
Marc Imhotep Cray MD
Case#9
71
A 67-year-old man presents to your clinic complaining of a 20-pound weight
loss, night sweats, and worsening malaise. He states that he has multiple
family members who have been diagnosed with cancer, but he is not sure
which type they have. He also says that he has not had any cancer screening,
including colonoscopies, because of “insurance issues.” A stool occult blood
test is positive, and a subsequent colonoscopy shows cancerous growths in
the sigmoid colon. A follow-up PET scan shows metastatic spread of his
cancer. After delivering the diagnosis to the patient and his family, you begin
to discuss initial treatment options. You note that one medication used to
treat metastatic colon cancer works by inhibiting the enzyme topoisomerase I.
What is the Medication?
Marc Imhotep Cray MD
Irinotecan
72
Similar Drugs
Topotecan
Mechanism of Action
Irinotecan acts by inhibiting topoisomerase I, which leads to excessive DNA
supercoiling and eventual DNA strand breakage and cell death
Clinical Uses
 Chemotherapeutic agent used in Tx of recurrent or metastatic solid tumors
(notably metastatic colon cancer for irinotecan and metastatic ovarian or
cervical cancer for topotecan)
 Other cancers treated with these agents include pancreatic, esophageal, and
small-cell lung cancer
Side Effects/AEs
Bone marrow suppression; GI upset.
Notes: Irinotecan and topotecan are cell cycle-specific drugs, active during
the S phase of cell cycle
Marc Imhotep Cray MD
Irinotecan(2)
73
Black Box Warnings: SEVERE DIARRHEA
Both early & late (after >24 hrs) forms of DIARRHEA may occur & can be SEVERE. Early
diarrhea may be accompanied by cholinergic symptoms & can be reduced by atropine.
Late diarrhea can be life-threatening due to dehydration, electrolyte imbalance or sepsis.
Late diarrhea should be treated promptly with loperamide. Patients with diarrhea should
be carefully monitored & given fluid, electrolytes, antibiotics if needed. Administration of
Irinotecan should be discontinued if severe diarrhea occurs.
Marc Imhotep Cray MD
Case#10
74
A 21-year-old man presents to the emergency room complaining of unsteady
gait, which he began to notice over the past week. Upon further questioning,
you learn that he was recently diagnosed with Hodgkin lymphoma and is
currently receiving chemotherapy for his disease. Physical examination is
significant for hyporeflexia in his lower extremities and right foot-drop.
Laboratory studies are unremarkable and, notably, his platelets, white blood
cell counts, and red blood cell counts are normal. As you prepare to admit the
patient to the neurology service, you wonder if his neurologic deficits are
related to his chemotherapeutic regimen.
What is the Medication?
Marc Imhotep Cray MD
Vincristine and Vinblastine
75
Mechanism of Action
Vincristine (VC) and Vinblastine (VB) bind to tubulin, a microtubular protein,
thereby leading to the depolymerization of mitotic spindle. Thus, cells are
unable to progress past metaphase during mitosis or meiosis, leading to
decreased cellular proliferation
Clinical Uses
 VC: Chemotherapeutic agent used in the treatment of ALL, lymphomas (part of the MOPP
protocol used to treat Hodgkin’s disease), Wilms’ tumor, and Ewing’s sarcoma.
 VB: Chemotherapeutic agent used to treat testicular cancer, breast cancer, non-small cell
lung cancer, and various lymphomas
Side Effects/AEs
 VC: Peripheral neuropathy; GI upset; alopecia; myelosuppression is rare and usually mild
when it does occur
 VB: Bone marrow suppression; GI upset; alopecia
Note: VC and VB are cell cycle-specific drugs, acting during M phase of cell
cycle
Marc Imhotep Cray MD
Case#11
76
A 43-year-old woman with metastatic breast cancer returns to your oncology
clinic. She has undergone combination chemotherapy for her metastatic disease
without a positive response. She is quite distraught and asks if her
chemotherapy can be changed. You inform her that there is a new therapy for
treatment of breast cancer in case combination chemotherapy fails. You
decide to switch her regimen to include this other medication, which acts by
interfering with the ability of the mitotic spindle to break down and thus
halting mitosis.
What is the Medication?
Marc Imhotep Cray MD
Paclitaxel
77
Similar Drugs
Docetaxel
Mechanism of Action
Paclitaxel binds to tubulin, a microtubular protein. The paclitaxel-tubulin
complex acts to promote stabilization and polymerization of mitotic
spindle leads to halt of mitosis during metaphase (anaphase cannot occur)
Clinical Uses
 Chemotherapeutic agent used in Tx of ovarian and breast cancer, small cell
cancer of lung, advanced Kaposi sarcoma, and head and neck cancers
 Paclitaxel has been used in past to coat coronary artery stents so as to
prevent restenosis of artery at site of stent
Side Effects/AEs
Hypersensitivity reactions (rash); bone marrow suppression; neurotoxicity
Marc Imhotep Cray MD
Paclitaxel (2)
78
Black Box Warnings: ANAPHYLAXIS & MYLEOSUPPRESSION
1.ANAPHYLAXIS & severe hypersensitivity reactions with dyspnea, hypotension, urticaria &
angioedema requiring treatment have occurred in 2-4% of patients. Fatal reactions have
occurred despite premedication.
2.ALL PATIENTS SHOULD BE PRETREATED with CORTICOSTEROIDS, DIPHENHYDRAMINE &
H2 ANTAGONISTS.
3.Paclitaxel should not be given to patients with solid tumors that have baseline neutrophil
counts of less than 1500 cells/mm3 & should NOT be given to patients with AIDS-related
Kaposi's sarcoma if their baseline neutrophil count is less than 1000 cells/mm3.
4.Frequent peripheral blood cell counts should be performed to monitor the occurrence of
bone marrow suppression (primarily neutropenia).
Marc Imhotep Cray MD
CHEMOTHERAPY IV: ANTI-TUMOR ANTIBIOTIC
AGENTS
79
1. The characteristic indications, mechanism of action,
mechanism of resistance and adverse effects of doxorubicin
and the other commonly used antibiotic and
anti-tumor agents.
2. The concept of multiple drug resistance and its effects on
effective chemotherapy.
3. The concept of cumulative toxicity and schedule
independent toxicity.
4. The concept of the hormonally-sensitive neoplasms and
their treatment.
5. The major side effects associated with antibiotic and anti-
tumor agents commonly used in chemotherapy.
Relevant Drugs:
Doxorubicin
Daunorubicin
Idarubicin
Dactinomycin
Bleomycin
Core Concepts and Learning Objectives
Marc Imhotep Cray MD
Case#13
80
A 14-year-old boy presents to your office for initiation of chemotherapy to
treat his acute lymphoblastic leukemia. You explain to the boy and his mother
that you will be using several different chemotherapy agents to treat his
disease. One of the agents you will be administering acts by blocking DNA and
RNA synthesis; however, this drug also causes the production of
oxygen free radicals, which can damage cardiac tissue when given at high
doses. You assure the patient that you will be carefully monitoring levels of
this drug so as to avoid cardiac toxicity if possible.
What is the Medication?
Marc Imhotep Cray MD
Anthracycline Antibiotics
81
Class of anthracyclines includes doxorubicin, daunorubicin, and idarubicin
Mechanism of Action
Anthracyclines block DNA and RNA synthesis both through direct steric
inhibition as well as by inhibiting topoisomerase II. They also cause
production of oxygen free radicals, which lead to membrane damage
Clinical Uses
 Doxorubicin: used in treating a variety of solid tumors (breast, ovary,
bladder, endometrium, stomach, lung) as well as in treating a variety of
hematologic malignancies (acute leukemia, lymphoma, multiple myeloma)
and in treating AIDS-related Kaposi sarcoma.
 Daunorubicin: used in treating acute leukemia (AML, ALL, CML) and
neuroblastoma
 Idarubicin: used in treating acute myeloid leukemia
Marc Imhotep Cray MD
Anthracyclines(2)
82
Side Effects/AEs
Cardiac toxicity; bone marrow suppression; GI distress; alopecia
Important Notes:
 The cardiotoxic effects of anthracyclines are caused, in part, by
production of oxygen free radicals within myocardial cells and
subsequent damage of myocyte.
 This adverse effect is cumulative, dose-dependent, and can be largely
avoided by keeping doses below a certain lifetime dose and by
administering dexrazoxane a compound that acts to decrease free
radical formation
Marc Imhotep Cray MD
Anthracyclines (3) doxorubicin, daunorubicin
83
Black Box Warnings: SEVERE MYELOSUPPRESSION & MYOCARDIAL TOXICITY
1.Daunorubicin for injection must be given into a rapidly flowing i.v. infusion. It must never be
given by i.m. or s.c. routes due to local tissue necrosis.
2.MYOCARDIAL TOXICITY manifested in its most severe form as potentially fatal CHF may occur
months to years after termination of therapy. The incidence increases after cumulative
doses exceed 400-550 mg/m2 in adults or 300 mg/m2 in children >2yo, or 10 mg/kg in children
<2yo.
3.SEVERE MYELOSUPPRESSION occurs at therapeutic doses, which may lead to infection or
hemorrhage.
4.dosage should be reduced in patients with impaired hepatic or renal function.
5.Daunorubicin should only be given in a facility where physicians & staff are capable of rapidly
responding to severe hemorrhagic conditions and/or overwhelming infection.
Marc Imhotep Cray MD
Case#14
84
A 2-year-old boy is brought to your oncology clinic after having been
recently diagnosed with Wilms tumor. Physical examination is significant for a
palpable left flank mass. You explain to the child’s mother that the best course
of treatment will include surgical excision of the tumor and chemotherapy. You
recommend the use of at least two drugs, one of which acts by interfering
with RNA synthesis through the inhibition of DNA dependent RNA polymerase.
Since this drug can also cause bone marrow suppression, the child’s blood cell
counts, especially his platelets and leukocytes, will have to be monitored
closely.
What is the Medication?
Marc Imhotep Cray MD
Dactinomycin
85
Similar Drugs
Plicamycin
Mechanism of Action
Dactinomycin intercalates betw. cytosine-guanine base pairs of DNA, thereby
acting to inhibit DNA-dependent RNA polymerase, and thereby impairing
both DNA and RNA synthesis.
Clinical Uses
Used for several solid tumors including Wilms tumor, Ewing sarcoma, and
metastatic testicular cancer. Also used in conjunction w MTX to treat
gestational choriocarcinoma
Side Effects/AEs
Bone marrow suppression; GI upset; skin abnormalities at sites that have
previously received radiation (“radiation recall”)
Note Plicamycin has a similar MOA as dactinomycin, but is primarily used for
Tx of some testicular cancers and Paget’s disease of bone
Marc Imhotep Cray MD
Case#15
86
A 69-year-old man presents to your clinic complaining of shortness of breath
that has become progressively worse over the past month. He tells you that he
was diagnosed with testicular cancer 4 months ago, for which he is currently
receiving chemotherapy. Physical examination is significant for an oxygen
saturation of 89% on room air and decreased air movement on lung
examination. When imaging studies reveal pulmonary fibrosis, you begin to
suspect that this patient’s symptoms are likely related to his chemotherapeutic
regimen.
What is the Medication?
Marc Imhotep Cray MD
Bleomycin
87
Mechanism of Action
Bleomycin binds to DNA and triggers the formation of oxygen free radicals.
The oxygen radicals then act to damage the DNA, leading to strand breaks
and inhibition of DNA synthesis.
Clinical Uses
Chemotherapeutic agent for the treatment of testicular tumors, squamous
cell carcinomas (head and neck, skin, penis, cervix), and Hodgkin’s
lymphoma.
Side Effects/AEs
Pulmonary fibrosis; allergic reactions (fever, anaphylaxis); mucositis; skin
changes
Note that myelosuppression is rare with bleomycin
Of Note Bleomycin is cell cycle-specific, thereby leading to accumulation of
cells in G2 phase of cell cycle
Marc Imhotep Cray MD
Bleomycin (2)
88
Black Box Warnings:
PULMONARY FIBROSIS is associated with bleomycin treatment. The most
common presentation is pneumonitis, progressing to pulmonary fibrosis. It
is more common in elderly patients receiving more than 400 units total
dose, but has been seen in younger patients & those treated with lower
doses. A severe IDIOSYNCRATIC REACTION of hypotension, mental
confusion, chills, fever & wheezing has also been reported in 1% of
lymphoma patients undergoing bleomycin treatment.
Marc Imhotep Cray MD
CHEMOTHERAPY V: TARGETED THERAPIES OF
CANCER MISCELLANEOUS AGENTS
89
1. The concept of “targeted cancer therapy”.
2. The role of tyrosine kinases as targets for cancer
therapy.
3. The potential benefits and toxicities of the
commercially available targeted therapies.
4. The uses and side effects of miscellaneous anti-
cancer agents.
Relevant Drugs
Imatinib mesylate
(Gleevec)
Erlotinib (Tarceva)
Cetuximab (Erbitux)
Trastuzumab (Herceptin)
Bevacizumab (Avastin)
Rituximab (Rituxin)
Asparaginase
Hydroxyurea
All-trans-retinoic acid
Arsenic trioxide
Marc Imhotep Cray MD
Case#16
90
A 56-year-old gentleman with a past medical history significant for hypertension
and tobacco use presents to your office complaining of hematuria. On physical
exam, you palpate a flank mass on the left. Ultrasound confirms an irregular, non-
cystic, vascular lesion in his left kidney. Follow-up biopsy confirms your suspicion
that the patient has a malignant growth in his kidney. You suspect that the
patient’s chemotherapy regimen will include an agent to inhibit angiogenesis.
What is the Medication?
Marc Imhotep Cray MD
Bevacizumab
91
Mechanism of Action
Bevacizumab is a monoclonal antibody specific for vascular endothelial
growth factor (VEGF). By blocking the activity of VEGF, bevacizumab prevents
angiogenesis, reducing vascular supply to tumors and retarding their growth.
Clinical Uses
 Chemotherapeutic agent used in Tx of solid tumors (ovarian, colorectal,
cervical, renal cell carcinoma)
 It is also used to treat dysregulated angiogenesis in wet age related macular
degeneration
Marc Imhotep Cray MD
Bevacizumab (2)
92
Side Effects/AEs
Arterial thromboembolism; hypertension; hemorrhage.
Related Notes
 Cetuximab is a monoclonal antibody against epidermal growth factor
receptor (EGFR). Inhibiting EGFR prevents cell growth and induces
apoptosis. It is used to treat solid cancers including head and neck cancer,
non-small cell lung cancer, and colorectal cancer. Side effects include an
acne like rash.
 Similar to bevacizumab, ranibizumab is a monoclonal antibody w activity
against VEGF; It is used for Tx of macular degeneration
Marc Imhotep Cray MD
Case#17
93
A 50-year-old woman returns to your oncology clinic for a follow-up visit. She
is currently on a multidrug regimen for metastatic breast cancer. She appears
to be responding well after the addition of a chemotherapeutic agent aimed
at the HER-2/neu receptors within her cancer cells. While she is in the office,
you schedule her for an imaging study to evaluate her heart. She asks why
this test needs to be done and you remind her that, although the new
medication is highly effective against her type of breast cancer, it can also
cause cardiac dysfunction in up to 10% of patients.
What is the Medication?
Marc Imhotep Cray MD
Trastuzumab
94
Mechanism of Action
Trastuzumab acts as a monoclonal antibody against the HER-2/neu (erb-B2)
receptor. The HER-2/neu receptor is involved in growth-promoting pathways
of the cell; 20% to 30% of breast cancers will overexpress HER-2/neu, thereby
contributing to uncontrolled cellular growth. Trastuzumab directly blocks
HER-2/neu signaling and also induces antibody-dependent cell-mediated
cytotoxicity.
Clinical Uses
Chemotherapeutic agent used to treat breast cancer and gastric cancer that
overexpress HER-2 receptor.
Side Effects/AEs
Cardiac toxicity; neutropenia; GI upset.
Marc Imhotep Cray MD
Case#18
95
A 49-year-old man presents to your oncology office for follow-up of his chronic
myelogenous leukemia, for which he is currently receiving chemotherapy. You
ask him whether he is experiencing any nausea, vomiting, or diarrhea, all of
which are common side effects of his medications. You also tell him that his
blood counts are at the low end of normal, which is expected since his
chemotherapeutic regimen can also cause bone marrow suppression. He was
biochemistry major in college, so he is interested in knowing how the drugs
that he is taking work. You explain to him that one of his medications acts by
inhibiting ribonucleotide reductase, an enzyme involved in making
deoxyribonucleotides, the building blocks for DNA.
What is the Medication?
Marc Imhotep Cray MD
Hydroxyurea
96
Mechanism of Action
Hydroxyurea inhibits ribonucleotide reductase, the enzyme responsible for
reducing ribonucleotides to deoxyribonucleotides. Inhibition of this enzyme
results in decreased DNA synthesis. Hydroxyurea has also been shown to
increase levels of fetal hemoglobin (Hb F), although exact mechanism for this
effect is unknown.
Clinical Uses
 Chemotherapeutic agent used to treat myeloproliferative disorders (e.g.,
CML, polycythemia vera) as well as melanoma
 Hydroxyurea has also been used in the treatment of sickle cell disease to
promote production of Hb F over hemoglobin S
Side Effects/AEs
Bone marrow suppression; GI upset.
Note Hydroxyurea is a cell cycle-specific agent, acting during S phase of cell
cycle
Marc Imhotep Cray MD
Case#19
97
A 64-year-old man with chronic myelogenous leukemia is being seen in
your oncology clinic. He has been treated with α-interferon and cytarabine
with no improvement. He asks you if there are any other options to treat his
disease. You tell him that you would like to try a different agent that acts by
inhibiting a tyrosine kinase on a mutated gene product, bcr-abl, which has
been shown to be associated with his disease.
What is the Medication?
Marc Imhotep Cray MD
Imatinib
98
Mechanism of Action
Imatinib acts as a competitive inhibitor of tyrosine kinase enzymes in abl, c-kit,
and PDGF-R (Platelet-derived growth factor receptors ), all genes involved in
cellular growth. By inhibiting tyrosine kinase enzymes, activation of these genes
is severely decreased, and cellular growth is slowed
Clinical Uses
Chemotherapeutic agent used to treat CML, GISTs (gastrointestinal stromal
tumor), and brain tumors
Side Effects/AEs
Fluid retention (edema); nausea; rash; heart failure
Important Note
CML is commonly assoc. w translocations betw. chromosomes 9 and 22, yielding
what is known as Philadelphia chromosome. The translocation produces a
fusion protein, bcr-abl, which is constitutively active By inhibiting tyrosine
kinase of abl portion of this mutated gene product wagents such as imatinib,
cellular growth of CML is decreased
Marc Imhotep Cray MD
Types of Chemotherapeutic Agents:
A Classification Summary
99
Marc Imhotep Cray MD
Alkylating Agents
100
Alkylating agents are most active in the resting phase of cell. These types
of drugs are cell-cycle non-specific.
Mustard gas derivatives: Mechlorethamine, Cyclophosphamide,
Chlorambucil, Melphalan, and Ifosfamide.
 Ethylenimines: Thiotepa and Hexamethylmelamine.
 Alkylsulfonates: Busulfan.
 Hydrazines and Triazines: Altretamine, Procarbazine, Dacarbazine and
Temozolomide.
 Nitrosureas: Carmustine, Lomustine and Streptozocin. Nitrosureas are
unique because, unlike most types of chemo treatments, they can cross
the blood-brain barrier. They can be useful in treating brain tumors.
 Metal salts: Carboplatin, Cisplatin, and Oxaliplatin
Marc Imhotep Cray MD
Plant Alkaloids
101
Plant alkaloids are ChemoTx agents derived made from certain types
of plants. Vinca alkaloids are made from periwinkle plant (catharanthus rosea).
Taxanes are made from bark of Pacific Yew tree (taxus). Vinca alkaloids and
taxanes are also known as antimicrotubule agents. The podophyllotoxins are
derived from the May apple plant. Camptothecan analogs are derived from
Asian "Happy Tree" (Camptotheca acuminata). Podophyllotoxins and
camptothecan analogs are also known as topoisomerase inhibitors, which are
used in certain types of chemotherapy.
The plant alkaloids are cell-cycle specific. This means they attack cells during
various phases of division
 Vinca alkaloids: Vincristine, Vinblastine and Vinorelbine.
 Taxanes: Paclitaxel and Docetaxel
 Podophyllotoxins: Etoposide and Tenisopide
 Camptothecan analogs: Irinotecan and Topotecan
Marc Imhotep Cray MD
Anti-tumor Antibiotics
102
Antitumor antibiotics are ChemoTx agents made from natural products
produced by species of soil fungus Streptomyces. These drugs act during
multiple phases of cell cycle and are considered cell-cycle specific. There are
several types of antitumor antibiotics:
•Anthracyclines: Doxorubicin, Daunorubicin, Epirubicin, Mitoxantrone, and
Idarubicin
•Chromomycins: Dactinomycin and Plicamycin
•Miscellaneous: Mitomycin and Bleomycin
Marc Imhotep Cray MD
Antimetabolites
103
Antimetabolites are types of ChemoTx that are very similar to normal
substances within cell. When cells incorporate these substances into
cellular metabolism, they are unable to divide. Antimetabolites are cell-
cycle specific. They attack cells at very specific phases in cycle.
Antimetabolites are classified according to substances with which they
interfere.
 Folic acid antagonist: Methotrexate
 Pyrimidine antagonist: 5-Fluorouracil, Foxuridine, Cytarabine,
Capecitabine, and Gemcitabine
 Purine antagonist: 6-Mercaptopurine and 6-Thioguanine
 Adenosine deaminase inhibitor: Cladribine, Fludarabine, Nelarabine and
Pentostatin
Marc Imhotep Cray MD
Topoisomerase Inhibitors
104
Toposiomerase inhibitors are types of ChemTx drugs that interfere with
action of topoisomerase enzymes (topoisomerase I and II). During process
of chemo Txs, topoisomerase enzymes control manipulation of structure of
DNA necessary for replication.
•Topoisomerase I inhibitors: Irinotecan, topotecan
•Topoisomerase II inhibitors: Amsacrine, etoposide, etoposide phosphate,
teniposide
Marc Imhotep Cray MD
Miscellaneous Antineoplastics
105
Several useful types of chemotherapy drugs are unique:
 Ribonucleotide reductase inhibitor: Hydroxyurea
 Adrenocortical steroid inhibitor: Mitotane
 Enzymes: Asparaginase and Pegaspargase
 Antimicrotubule agent: Estramustine
 Retinoids: Bexarotene, Isotretinoin, Tretinoin (ATRA)
 Beyond aforementioned types of ChemoTx agents, many other types of
chemo treatments exist, such as targeted therapy, immunotherapy, and
hormone therapy.
Sources:
 http://chemocare.com/chemotherapy/what-is-chemotherapy/types-of-chemotherapy.aspx
 http://tmedweb.tulane.edu/pharmwiki/doku.php/cancer_chemotherapy
106
THE END
See next slide for links to tools and resources for further study.
Marc Imhotep Cray MD
Further Study
107
Cloud Folder: CA ChemoTx Ppts, Notes, SDL Tutorials and Video

Más contenido relacionado

La actualidad más candente

32.tuberculosis and leprosy
32.tuberculosis and leprosy 32.tuberculosis and leprosy
32.tuberculosis and leprosy Dr.Manish Kumar
 
NurseReview.Org - Antimalarials Updates (pharmacology tutorial)
NurseReview.Org - Antimalarials Updates (pharmacology tutorial)NurseReview.Org - Antimalarials Updates (pharmacology tutorial)
NurseReview.Org - Antimalarials Updates (pharmacology tutorial)jben501
 
cancer chemotherapy
cancer chemotherapycancer chemotherapy
cancer chemotherapyNaser Tadvi
 
Cancer Chemotherapy
Cancer ChemotherapyCancer Chemotherapy
Cancer Chemotherapyazsyed
 
Antiamoebic and antiprotozoal drugs - drdhriti
Antiamoebic and antiprotozoal drugs - drdhritiAntiamoebic and antiprotozoal drugs - drdhriti
Antiamoebic and antiprotozoal drugs - drdhritihttp://neigrihms.gov.in/
 
Anticancer drugs 4 cytotoxic drugs and antibiotics
Anticancer drugs 4 cytotoxic drugs and antibioticsAnticancer drugs 4 cytotoxic drugs and antibiotics
Anticancer drugs 4 cytotoxic drugs and antibioticsSubramani Parasuraman
 
Overview and classification of chemotherapeutic agents and theory
Overview and classification of  chemotherapeutic agents and theoryOverview and classification of  chemotherapeutic agents and theory
Overview and classification of chemotherapeutic agents and theorySaurabh Gupta
 
Adverse drug reactions of tetracycline antibiotics
Adverse drug reactions of tetracycline antibioticsAdverse drug reactions of tetracycline antibiotics
Adverse drug reactions of tetracycline antibioticsManiz Joshi
 
Introduction to antimicrobials
Introduction to antimicrobialsIntroduction to antimicrobials
Introduction to antimicrobialsZainab&Sons
 
Introduction to chemotherapy of cancer
Introduction to chemotherapy of cancerIntroduction to chemotherapy of cancer
Introduction to chemotherapy of cancerSubramani Parasuraman
 

La actualidad más candente (20)

10.ANTICANCER DRUGS
10.ANTICANCER DRUGS10.ANTICANCER DRUGS
10.ANTICANCER DRUGS
 
Chemotherapy
ChemotherapyChemotherapy
Chemotherapy
 
32.tuberculosis and leprosy
32.tuberculosis and leprosy 32.tuberculosis and leprosy
32.tuberculosis and leprosy
 
NurseReview.Org - Antimalarials Updates (pharmacology tutorial)
NurseReview.Org - Antimalarials Updates (pharmacology tutorial)NurseReview.Org - Antimalarials Updates (pharmacology tutorial)
NurseReview.Org - Antimalarials Updates (pharmacology tutorial)
 
cancer chemotherapy
cancer chemotherapycancer chemotherapy
cancer chemotherapy
 
Anti Cancer Drugs
Anti  Cancer DrugsAnti  Cancer Drugs
Anti Cancer Drugs
 
Cancer Chemotherapy
Cancer ChemotherapyCancer Chemotherapy
Cancer Chemotherapy
 
Chemotherapy of cancer
Chemotherapy of cancerChemotherapy of cancer
Chemotherapy of cancer
 
Antiamoebic and antiprotozoal drugs - drdhriti
Antiamoebic and antiprotozoal drugs - drdhritiAntiamoebic and antiprotozoal drugs - drdhriti
Antiamoebic and antiprotozoal drugs - drdhriti
 
Anticancer drugs 4 cytotoxic drugs and antibiotics
Anticancer drugs 4 cytotoxic drugs and antibioticsAnticancer drugs 4 cytotoxic drugs and antibiotics
Anticancer drugs 4 cytotoxic drugs and antibiotics
 
Overview and classification of chemotherapeutic agents and theory
Overview and classification of  chemotherapeutic agents and theoryOverview and classification of  chemotherapeutic agents and theory
Overview and classification of chemotherapeutic agents and theory
 
Adverse drug reactions of tetracycline antibiotics
Adverse drug reactions of tetracycline antibioticsAdverse drug reactions of tetracycline antibiotics
Adverse drug reactions of tetracycline antibiotics
 
How to Manage Chemobrain
How to Manage ChemobrainHow to Manage Chemobrain
How to Manage Chemobrain
 
Antiprotozoal druga their mode of action
Antiprotozoal druga their mode of actionAntiprotozoal druga their mode of action
Antiprotozoal druga their mode of action
 
Introduction to antimicrobials
Introduction to antimicrobialsIntroduction to antimicrobials
Introduction to antimicrobials
 
Introduction to chemotherapy of cancer
Introduction to chemotherapy of cancerIntroduction to chemotherapy of cancer
Introduction to chemotherapy of cancer
 
Antimalarial drugs
Antimalarial drugsAntimalarial drugs
Antimalarial drugs
 
Anti tubercular drugs
Anti tubercular drugsAnti tubercular drugs
Anti tubercular drugs
 
Anticancer
AnticancerAnticancer
Anticancer
 
Anti HIV drugs part 2
Anti HIV drugs part 2Anti HIV drugs part 2
Anti HIV drugs part 2
 

Similar a Drugs Used in Neoplastic Disorders

01-Ca Intro_2016.pdf pharmacy student ca
01-Ca Intro_2016.pdf pharmacy student ca01-Ca Intro_2016.pdf pharmacy student ca
01-Ca Intro_2016.pdf pharmacy student cajiropharm
 
1. Chemo in the home
1. Chemo in the home1. Chemo in the home
1. Chemo in the homeChartwellPA
 
Chemotherapeutic drugs123
Chemotherapeutic drugs123Chemotherapeutic drugs123
Chemotherapeutic drugs123nourahmed98
 
Chp no 3 ANTI-NEOPLASTIC DRUGS.pptx
Chp no 3 ANTI-NEOPLASTIC DRUGS.pptxChp no 3 ANTI-NEOPLASTIC DRUGS.pptx
Chp no 3 ANTI-NEOPLASTIC DRUGS.pptxMahnoorFatima92
 
neoplastic_disorders.pptx
neoplastic_disorders.pptxneoplastic_disorders.pptx
neoplastic_disorders.pptxsuhaj2
 
AntineoplasticAgentsby KEMISA. HASSEN ZAINABU
AntineoplasticAgentsby KEMISA. HASSEN ZAINABU AntineoplasticAgentsby KEMISA. HASSEN ZAINABU
AntineoplasticAgentsby KEMISA. HASSEN ZAINABU HASSENZAINABUKEMISA
 
General principles in chemotherapy of cancer
General principles in chemotherapy of cancerGeneral principles in chemotherapy of cancer
General principles in chemotherapy of cancerKLE COLLEGE OF PHARMACY
 
9 tumor+chemotherapy english -+version imp
9   tumor+chemotherapy english -+version imp9   tumor+chemotherapy english -+version imp
9 tumor+chemotherapy english -+version impSumit Prajapati
 
9 Tumor+Chemotherapy English +Version Imp
9   Tumor+Chemotherapy English  +Version Imp9   Tumor+Chemotherapy English  +Version Imp
9 Tumor+Chemotherapy English +Version ImpSumit Prajapati
 
Cancer chemotherapy for medical students
Cancer chemotherapy for medical studentsCancer chemotherapy for medical students
Cancer chemotherapy for medical studentstaklo simeneh
 
Metronomic Chemotherapy
Metronomic ChemotherapyMetronomic Chemotherapy
Metronomic ChemotherapySonali Karekar
 
Chemotherapeutic Drugs
Chemotherapeutic DrugsChemotherapeutic Drugs
Chemotherapeutic DrugsYogeshIJTSRD
 
Chemotherapy Friends or Foe to Cancer Immunotherapy by Prof. Mohamed L. Salem
Chemotherapy Friends or Foe to Cancer Immunotherapy by Prof. Mohamed L. SalemChemotherapy Friends or Foe to Cancer Immunotherapy by Prof. Mohamed L. Salem
Chemotherapy Friends or Foe to Cancer Immunotherapy by Prof. Mohamed L. SalemProf. Mohamed Labib Salem
 

Similar a Drugs Used in Neoplastic Disorders (20)

01-Ca Intro_2016.pdf pharmacy student ca
01-Ca Intro_2016.pdf pharmacy student ca01-Ca Intro_2016.pdf pharmacy student ca
01-Ca Intro_2016.pdf pharmacy student ca
 
Mesotheliomahelp org treatment_chemotherapy
Mesotheliomahelp org treatment_chemotherapyMesotheliomahelp org treatment_chemotherapy
Mesotheliomahelp org treatment_chemotherapy
 
1. Chemo in the home
1. Chemo in the home1. Chemo in the home
1. Chemo in the home
 
Anti cancer drugs
Anti cancer drugsAnti cancer drugs
Anti cancer drugs
 
Chemotherapeutic drugs123
Chemotherapeutic drugs123Chemotherapeutic drugs123
Chemotherapeutic drugs123
 
Chp no 3 ANTI-NEOPLASTIC DRUGS.pptx
Chp no 3 ANTI-NEOPLASTIC DRUGS.pptxChp no 3 ANTI-NEOPLASTIC DRUGS.pptx
Chp no 3 ANTI-NEOPLASTIC DRUGS.pptx
 
neoplastic_disorders.pptx
neoplastic_disorders.pptxneoplastic_disorders.pptx
neoplastic_disorders.pptx
 
AntineoplasticAgentsby KEMISA. HASSEN ZAINABU
AntineoplasticAgentsby KEMISA. HASSEN ZAINABU AntineoplasticAgentsby KEMISA. HASSEN ZAINABU
AntineoplasticAgentsby KEMISA. HASSEN ZAINABU
 
Cancer Treatment
Cancer TreatmentCancer Treatment
Cancer Treatment
 
General principles in chemotherapy of cancer
General principles in chemotherapy of cancerGeneral principles in chemotherapy of cancer
General principles in chemotherapy of cancer
 
Cance1
Cance1Cance1
Cance1
 
9 tumor+chemotherapy english -+version imp
9   tumor+chemotherapy english -+version imp9   tumor+chemotherapy english -+version imp
9 tumor+chemotherapy english -+version imp
 
9 Tumor+Chemotherapy English +Version Imp
9   Tumor+Chemotherapy English  +Version Imp9   Tumor+Chemotherapy English  +Version Imp
9 Tumor+Chemotherapy English +Version Imp
 
Cancer chemotherapy for medical students
Cancer chemotherapy for medical studentsCancer chemotherapy for medical students
Cancer chemotherapy for medical students
 
Metronomic Chemotherapy
Metronomic ChemotherapyMetronomic Chemotherapy
Metronomic Chemotherapy
 
Chemotherapeutic Drugs
Chemotherapeutic DrugsChemotherapeutic Drugs
Chemotherapeutic Drugs
 
Chemotherapy Friends or Foe to Cancer Immunotherapy by Prof. Mohamed L. Salem
Chemotherapy Friends or Foe to Cancer Immunotherapy by Prof. Mohamed L. SalemChemotherapy Friends or Foe to Cancer Immunotherapy by Prof. Mohamed L. Salem
Chemotherapy Friends or Foe to Cancer Immunotherapy by Prof. Mohamed L. Salem
 
Cancer chemotherapy part i
Cancer chemotherapy part iCancer chemotherapy part i
Cancer chemotherapy part i
 
Systemic Therapy for Metastatic Disease
Systemic Therapy for Metastatic DiseaseSystemic Therapy for Metastatic Disease
Systemic Therapy for Metastatic Disease
 
Anti cancer drugs
Anti  cancer drugs Anti  cancer drugs
Anti cancer drugs
 

Más de Imhotep Virtual Medical School

Oncologic Pathology_A Case-based Organ Systems Review (USMLE Step 1)
Oncologic Pathology_A Case-based Organ Systems Review (USMLE Step 1)Oncologic Pathology_A Case-based Organ Systems Review (USMLE Step 1)
Oncologic Pathology_A Case-based Organ Systems Review (USMLE Step 1)Imhotep Virtual Medical School
 
Reproductive System Pathology_FM Breast and FM Reproductive Systems
Reproductive System Pathology_FM Breast and FM Reproductive SystemsReproductive System Pathology_FM Breast and FM Reproductive Systems
Reproductive System Pathology_FM Breast and FM Reproductive SystemsImhotep Virtual Medical School
 
Reproductive System Pathology_Male Reproductive Systems
Reproductive System Pathology_Male Reproductive SystemsReproductive System Pathology_Male Reproductive Systems
Reproductive System Pathology_Male Reproductive SystemsImhotep Virtual Medical School
 
Nervous System Pathology_A Case-based Learning Approach
Nervous System Pathology_A Case-based Learning ApproachNervous System Pathology_A Case-based Learning Approach
Nervous System Pathology_A Case-based Learning ApproachImhotep Virtual Medical School
 
CVS Function, Regulation of the Heart and Overview of Therapeutic Goals in CV...
CVS Function, Regulation of the Heart and Overview of Therapeutic Goals in CV...CVS Function, Regulation of the Heart and Overview of Therapeutic Goals in CV...
CVS Function, Regulation of the Heart and Overview of Therapeutic Goals in CV...Imhotep Virtual Medical School
 
Cardiovascular Pathology Case-based_Gross and Microscopic
Cardiovascular Pathology Case-based_Gross and MicroscopicCardiovascular Pathology Case-based_Gross and Microscopic
Cardiovascular Pathology Case-based_Gross and MicroscopicImhotep Virtual Medical School
 
Clinical Pharmacology for Medical Students_USMLE Step 1 & 2 Review
Clinical Pharmacology for Medical Students_USMLE Step 1 & 2 ReviewClinical Pharmacology for Medical Students_USMLE Step 1 & 2 Review
Clinical Pharmacology for Medical Students_USMLE Step 1 & 2 ReviewImhotep Virtual Medical School
 
Make the Dx_ A Case-based Intro to Select Cardiovascular and Respiratory Dise...
Make the Dx_ A Case-based Intro to Select Cardiovascular and Respiratory Dise...Make the Dx_ A Case-based Intro to Select Cardiovascular and Respiratory Dise...
Make the Dx_ A Case-based Intro to Select Cardiovascular and Respiratory Dise...Imhotep Virtual Medical School
 
Myocardial infarction_ Causes, Symptoms, Diagnosis, Treatment, and Pathology
Myocardial infarction_ Causes, Symptoms, Diagnosis, Treatment, and PathologyMyocardial infarction_ Causes, Symptoms, Diagnosis, Treatment, and Pathology
Myocardial infarction_ Causes, Symptoms, Diagnosis, Treatment, and PathologyImhotep Virtual Medical School
 
Autonomic Nervous System Physiology and Pharmacology_Overview| Review of ANS
Autonomic Nervous System Physiology and Pharmacology_Overview| Review of ANSAutonomic Nervous System Physiology and Pharmacology_Overview| Review of ANS
Autonomic Nervous System Physiology and Pharmacology_Overview| Review of ANSImhotep Virtual Medical School
 
Evidence-Based Physical Diagnosis_Lect. 1_ What is Evidence-Based Physical Di...
Evidence-Based Physical Diagnosis_Lect. 1_ What is Evidence-Based Physical Di...Evidence-Based Physical Diagnosis_Lect. 1_ What is Evidence-Based Physical Di...
Evidence-Based Physical Diagnosis_Lect. 1_ What is Evidence-Based Physical Di...Imhotep Virtual Medical School
 

Más de Imhotep Virtual Medical School (20)

Oncologic Pathology_A Case-based Organ Systems Review (USMLE Step 1)
Oncologic Pathology_A Case-based Organ Systems Review (USMLE Step 1)Oncologic Pathology_A Case-based Organ Systems Review (USMLE Step 1)
Oncologic Pathology_A Case-based Organ Systems Review (USMLE Step 1)
 
Pathology and Pathophysiology of Shock
Pathology and Pathophysiology of ShockPathology and Pathophysiology of Shock
Pathology and Pathophysiology of Shock
 
Drugs Used In Disorders of the Reproductive System
Drugs Used In Disorders of the Reproductive SystemDrugs Used In Disorders of the Reproductive System
Drugs Used In Disorders of the Reproductive System
 
Reproductive System Pathology_FM Breast and FM Reproductive Systems
Reproductive System Pathology_FM Breast and FM Reproductive SystemsReproductive System Pathology_FM Breast and FM Reproductive Systems
Reproductive System Pathology_FM Breast and FM Reproductive Systems
 
Reproductive System Pathology_Male Reproductive Systems
Reproductive System Pathology_Male Reproductive SystemsReproductive System Pathology_Male Reproductive Systems
Reproductive System Pathology_Male Reproductive Systems
 
Nervous System Pathology_A Case-based Learning Approach
Nervous System Pathology_A Case-based Learning ApproachNervous System Pathology_A Case-based Learning Approach
Nervous System Pathology_A Case-based Learning Approach
 
CVS Function, Regulation of the Heart and Overview of Therapeutic Goals in CV...
CVS Function, Regulation of the Heart and Overview of Therapeutic Goals in CV...CVS Function, Regulation of the Heart and Overview of Therapeutic Goals in CV...
CVS Function, Regulation of the Heart and Overview of Therapeutic Goals in CV...
 
Cardiovascular Pathology Case-based_Gross and Microscopic
Cardiovascular Pathology Case-based_Gross and MicroscopicCardiovascular Pathology Case-based_Gross and Microscopic
Cardiovascular Pathology Case-based_Gross and Microscopic
 
HIV / AIDS Pathology
HIV / AIDS PathologyHIV / AIDS Pathology
HIV / AIDS Pathology
 
Sepsis & Septic Shock
Sepsis & Septic ShockSepsis & Septic Shock
Sepsis & Septic Shock
 
Drugs Used in infectious Disease_Antibiotics
Drugs Used in infectious Disease_AntibioticsDrugs Used in infectious Disease_Antibiotics
Drugs Used in infectious Disease_Antibiotics
 
Hematopoietic and Lymphoid Systems Pathology
Hematopoietic and Lymphoid Systems  PathologyHematopoietic and Lymphoid Systems  Pathology
Hematopoietic and Lymphoid Systems Pathology
 
Neoplasia & Oncologic Pathology
Neoplasia & Oncologic PathologyNeoplasia & Oncologic Pathology
Neoplasia & Oncologic Pathology
 
Clinical Pharmacology for Medical Students_USMLE Step 1 & 2 Review
Clinical Pharmacology for Medical Students_USMLE Step 1 & 2 ReviewClinical Pharmacology for Medical Students_USMLE Step 1 & 2 Review
Clinical Pharmacology for Medical Students_USMLE Step 1 & 2 Review
 
Make the Dx_ A Case-based Intro to Select Cardiovascular and Respiratory Dise...
Make the Dx_ A Case-based Intro to Select Cardiovascular and Respiratory Dise...Make the Dx_ A Case-based Intro to Select Cardiovascular and Respiratory Dise...
Make the Dx_ A Case-based Intro to Select Cardiovascular and Respiratory Dise...
 
Myocardial infarction_ Causes, Symptoms, Diagnosis, Treatment, and Pathology
Myocardial infarction_ Causes, Symptoms, Diagnosis, Treatment, and PathologyMyocardial infarction_ Causes, Symptoms, Diagnosis, Treatment, and Pathology
Myocardial infarction_ Causes, Symptoms, Diagnosis, Treatment, and Pathology
 
Basic CXR Interpretation_Diagnostic Radiographs
Basic CXR Interpretation_Diagnostic RadiographsBasic CXR Interpretation_Diagnostic Radiographs
Basic CXR Interpretation_Diagnostic Radiographs
 
Electrocardiogram (ECG) Interpretation_Module 1 of 2
Electrocardiogram (ECG) Interpretation_Module 1 of 2Electrocardiogram (ECG) Interpretation_Module 1 of 2
Electrocardiogram (ECG) Interpretation_Module 1 of 2
 
Autonomic Nervous System Physiology and Pharmacology_Overview| Review of ANS
Autonomic Nervous System Physiology and Pharmacology_Overview| Review of ANSAutonomic Nervous System Physiology and Pharmacology_Overview| Review of ANS
Autonomic Nervous System Physiology and Pharmacology_Overview| Review of ANS
 
Evidence-Based Physical Diagnosis_Lect. 1_ What is Evidence-Based Physical Di...
Evidence-Based Physical Diagnosis_Lect. 1_ What is Evidence-Based Physical Di...Evidence-Based Physical Diagnosis_Lect. 1_ What is Evidence-Based Physical Di...
Evidence-Based Physical Diagnosis_Lect. 1_ What is Evidence-Based Physical Di...
 

Último

Premium Call Girls In Jaipur {8445551418} ❤️VVIP SEEMA Call Girl in Jaipur Ra...
Premium Call Girls In Jaipur {8445551418} ❤️VVIP SEEMA Call Girl in Jaipur Ra...Premium Call Girls In Jaipur {8445551418} ❤️VVIP SEEMA Call Girl in Jaipur Ra...
Premium Call Girls In Jaipur {8445551418} ❤️VVIP SEEMA Call Girl in Jaipur Ra...parulsinha
 
Top Rated Call Girls Kerala ☎ 8250092165👄 Delivery in 20 Mins Near Me
Top Rated Call Girls Kerala ☎ 8250092165👄 Delivery in 20 Mins Near MeTop Rated Call Girls Kerala ☎ 8250092165👄 Delivery in 20 Mins Near Me
Top Rated Call Girls Kerala ☎ 8250092165👄 Delivery in 20 Mins Near Mechennailover
 
Top Rated Pune Call Girls (DIPAL) ⟟ 8250077686 ⟟ Call Me For Genuine Sex Serv...
Top Rated Pune Call Girls (DIPAL) ⟟ 8250077686 ⟟ Call Me For Genuine Sex Serv...Top Rated Pune Call Girls (DIPAL) ⟟ 8250077686 ⟟ Call Me For Genuine Sex Serv...
Top Rated Pune Call Girls (DIPAL) ⟟ 8250077686 ⟟ Call Me For Genuine Sex Serv...Dipal Arora
 
Call Girls Rishikesh Just Call 9667172968 Top Class Call Girl Service Available
Call Girls Rishikesh Just Call 9667172968 Top Class Call Girl Service AvailableCall Girls Rishikesh Just Call 9667172968 Top Class Call Girl Service Available
Call Girls Rishikesh Just Call 9667172968 Top Class Call Girl Service Availableperfect solution
 
Low Rate Call Girls Bangalore {7304373326} ❤️VVIP NISHA Call Girls in Bangalo...
Low Rate Call Girls Bangalore {7304373326} ❤️VVIP NISHA Call Girls in Bangalo...Low Rate Call Girls Bangalore {7304373326} ❤️VVIP NISHA Call Girls in Bangalo...
Low Rate Call Girls Bangalore {7304373326} ❤️VVIP NISHA Call Girls in Bangalo...Sheetaleventcompany
 
Dehradun Call Girls Service {8854095900} ❤️VVIP ROCKY Call Girl in Dehradun U...
Dehradun Call Girls Service {8854095900} ❤️VVIP ROCKY Call Girl in Dehradun U...Dehradun Call Girls Service {8854095900} ❤️VVIP ROCKY Call Girl in Dehradun U...
Dehradun Call Girls Service {8854095900} ❤️VVIP ROCKY Call Girl in Dehradun U...Sheetaleventcompany
 
Coimbatore Call Girls in Coimbatore 7427069034 genuine Escort Service Girl 10...
Coimbatore Call Girls in Coimbatore 7427069034 genuine Escort Service Girl 10...Coimbatore Call Girls in Coimbatore 7427069034 genuine Escort Service Girl 10...
Coimbatore Call Girls in Coimbatore 7427069034 genuine Escort Service Girl 10...chennailover
 
Call Girls Service Jaipur {8445551418} ❤️VVIP BHAWNA Call Girl in Jaipur Raja...
Call Girls Service Jaipur {8445551418} ❤️VVIP BHAWNA Call Girl in Jaipur Raja...Call Girls Service Jaipur {8445551418} ❤️VVIP BHAWNA Call Girl in Jaipur Raja...
Call Girls Service Jaipur {8445551418} ❤️VVIP BHAWNA Call Girl in Jaipur Raja...parulsinha
 
Kollam call girls Mallu aunty service 7877702510
Kollam call girls Mallu aunty service 7877702510Kollam call girls Mallu aunty service 7877702510
Kollam call girls Mallu aunty service 7877702510Vipesco
 
VIP Hyderabad Call Girls Bahadurpally 7877925207 ₹5000 To 25K With AC Room 💚😋
VIP Hyderabad Call Girls Bahadurpally 7877925207 ₹5000 To 25K With AC Room 💚😋VIP Hyderabad Call Girls Bahadurpally 7877925207 ₹5000 To 25K With AC Room 💚😋
VIP Hyderabad Call Girls Bahadurpally 7877925207 ₹5000 To 25K With AC Room 💚😋TANUJA PANDEY
 
Call Girls Hyderabad Just Call 8250077686 Top Class Call Girl Service Available
Call Girls Hyderabad Just Call 8250077686 Top Class Call Girl Service AvailableCall Girls Hyderabad Just Call 8250077686 Top Class Call Girl Service Available
Call Girls Hyderabad Just Call 8250077686 Top Class Call Girl Service AvailableDipal Arora
 
Mumbai ] (Call Girls) in Mumbai 10k @ I'm VIP Independent Escorts Girls 98333...
Mumbai ] (Call Girls) in Mumbai 10k @ I'm VIP Independent Escorts Girls 98333...Mumbai ] (Call Girls) in Mumbai 10k @ I'm VIP Independent Escorts Girls 98333...
Mumbai ] (Call Girls) in Mumbai 10k @ I'm VIP Independent Escorts Girls 98333...Ishani Gupta
 
Jogeshwari ! Call Girls Service Mumbai - 450+ Call Girl Cash Payment 90042684...
Jogeshwari ! Call Girls Service Mumbai - 450+ Call Girl Cash Payment 90042684...Jogeshwari ! Call Girls Service Mumbai - 450+ Call Girl Cash Payment 90042684...
Jogeshwari ! Call Girls Service Mumbai - 450+ Call Girl Cash Payment 90042684...Anamika Rawat
 
Call Girls Mumbai Just Call 8250077686 Top Class Call Girl Service Available
Call Girls Mumbai Just Call 8250077686 Top Class Call Girl Service AvailableCall Girls Mumbai Just Call 8250077686 Top Class Call Girl Service Available
Call Girls Mumbai Just Call 8250077686 Top Class Call Girl Service AvailableDipal Arora
 
Coimbatore Call Girls in Thudiyalur : 7427069034 High Profile Model Escorts |...
Coimbatore Call Girls in Thudiyalur : 7427069034 High Profile Model Escorts |...Coimbatore Call Girls in Thudiyalur : 7427069034 High Profile Model Escorts |...
Coimbatore Call Girls in Thudiyalur : 7427069034 High Profile Model Escorts |...chennailover
 
8980367676 Call Girls In Ahmedabad Escort Service Available 24×7 In Ahmedabad
8980367676 Call Girls In Ahmedabad Escort Service Available 24×7 In Ahmedabad8980367676 Call Girls In Ahmedabad Escort Service Available 24×7 In Ahmedabad
8980367676 Call Girls In Ahmedabad Escort Service Available 24×7 In AhmedabadGENUINE ESCORT AGENCY
 
Call Girls Coimbatore Just Call 8250077686 Top Class Call Girl Service Available
Call Girls Coimbatore Just Call 8250077686 Top Class Call Girl Service AvailableCall Girls Coimbatore Just Call 8250077686 Top Class Call Girl Service Available
Call Girls Coimbatore Just Call 8250077686 Top Class Call Girl Service AvailableDipal Arora
 
9630942363 Genuine Call Girls In Ahmedabad Gujarat Call Girls Service
9630942363 Genuine Call Girls In Ahmedabad Gujarat Call Girls Service9630942363 Genuine Call Girls In Ahmedabad Gujarat Call Girls Service
9630942363 Genuine Call Girls In Ahmedabad Gujarat Call Girls ServiceGENUINE ESCORT AGENCY
 
Call Girls in Delhi Triveni Complex Escort Service(🔝))/WhatsApp 97111⇛47426
Call Girls in Delhi Triveni Complex Escort Service(🔝))/WhatsApp 97111⇛47426Call Girls in Delhi Triveni Complex Escort Service(🔝))/WhatsApp 97111⇛47426
Call Girls in Delhi Triveni Complex Escort Service(🔝))/WhatsApp 97111⇛47426jennyeacort
 

Último (20)

Premium Call Girls In Jaipur {8445551418} ❤️VVIP SEEMA Call Girl in Jaipur Ra...
Premium Call Girls In Jaipur {8445551418} ❤️VVIP SEEMA Call Girl in Jaipur Ra...Premium Call Girls In Jaipur {8445551418} ❤️VVIP SEEMA Call Girl in Jaipur Ra...
Premium Call Girls In Jaipur {8445551418} ❤️VVIP SEEMA Call Girl in Jaipur Ra...
 
Top Rated Call Girls Kerala ☎ 8250092165👄 Delivery in 20 Mins Near Me
Top Rated Call Girls Kerala ☎ 8250092165👄 Delivery in 20 Mins Near MeTop Rated Call Girls Kerala ☎ 8250092165👄 Delivery in 20 Mins Near Me
Top Rated Call Girls Kerala ☎ 8250092165👄 Delivery in 20 Mins Near Me
 
Top Rated Pune Call Girls (DIPAL) ⟟ 8250077686 ⟟ Call Me For Genuine Sex Serv...
Top Rated Pune Call Girls (DIPAL) ⟟ 8250077686 ⟟ Call Me For Genuine Sex Serv...Top Rated Pune Call Girls (DIPAL) ⟟ 8250077686 ⟟ Call Me For Genuine Sex Serv...
Top Rated Pune Call Girls (DIPAL) ⟟ 8250077686 ⟟ Call Me For Genuine Sex Serv...
 
Call Girls Rishikesh Just Call 9667172968 Top Class Call Girl Service Available
Call Girls Rishikesh Just Call 9667172968 Top Class Call Girl Service AvailableCall Girls Rishikesh Just Call 9667172968 Top Class Call Girl Service Available
Call Girls Rishikesh Just Call 9667172968 Top Class Call Girl Service Available
 
Low Rate Call Girls Bangalore {7304373326} ❤️VVIP NISHA Call Girls in Bangalo...
Low Rate Call Girls Bangalore {7304373326} ❤️VVIP NISHA Call Girls in Bangalo...Low Rate Call Girls Bangalore {7304373326} ❤️VVIP NISHA Call Girls in Bangalo...
Low Rate Call Girls Bangalore {7304373326} ❤️VVIP NISHA Call Girls in Bangalo...
 
Dehradun Call Girls Service {8854095900} ❤️VVIP ROCKY Call Girl in Dehradun U...
Dehradun Call Girls Service {8854095900} ❤️VVIP ROCKY Call Girl in Dehradun U...Dehradun Call Girls Service {8854095900} ❤️VVIP ROCKY Call Girl in Dehradun U...
Dehradun Call Girls Service {8854095900} ❤️VVIP ROCKY Call Girl in Dehradun U...
 
Coimbatore Call Girls in Coimbatore 7427069034 genuine Escort Service Girl 10...
Coimbatore Call Girls in Coimbatore 7427069034 genuine Escort Service Girl 10...Coimbatore Call Girls in Coimbatore 7427069034 genuine Escort Service Girl 10...
Coimbatore Call Girls in Coimbatore 7427069034 genuine Escort Service Girl 10...
 
Call Girls Service Jaipur {8445551418} ❤️VVIP BHAWNA Call Girl in Jaipur Raja...
Call Girls Service Jaipur {8445551418} ❤️VVIP BHAWNA Call Girl in Jaipur Raja...Call Girls Service Jaipur {8445551418} ❤️VVIP BHAWNA Call Girl in Jaipur Raja...
Call Girls Service Jaipur {8445551418} ❤️VVIP BHAWNA Call Girl in Jaipur Raja...
 
Kollam call girls Mallu aunty service 7877702510
Kollam call girls Mallu aunty service 7877702510Kollam call girls Mallu aunty service 7877702510
Kollam call girls Mallu aunty service 7877702510
 
VIP Hyderabad Call Girls Bahadurpally 7877925207 ₹5000 To 25K With AC Room 💚😋
VIP Hyderabad Call Girls Bahadurpally 7877925207 ₹5000 To 25K With AC Room 💚😋VIP Hyderabad Call Girls Bahadurpally 7877925207 ₹5000 To 25K With AC Room 💚😋
VIP Hyderabad Call Girls Bahadurpally 7877925207 ₹5000 To 25K With AC Room 💚😋
 
Call Girls Hyderabad Just Call 8250077686 Top Class Call Girl Service Available
Call Girls Hyderabad Just Call 8250077686 Top Class Call Girl Service AvailableCall Girls Hyderabad Just Call 8250077686 Top Class Call Girl Service Available
Call Girls Hyderabad Just Call 8250077686 Top Class Call Girl Service Available
 
Mumbai ] (Call Girls) in Mumbai 10k @ I'm VIP Independent Escorts Girls 98333...
Mumbai ] (Call Girls) in Mumbai 10k @ I'm VIP Independent Escorts Girls 98333...Mumbai ] (Call Girls) in Mumbai 10k @ I'm VIP Independent Escorts Girls 98333...
Mumbai ] (Call Girls) in Mumbai 10k @ I'm VIP Independent Escorts Girls 98333...
 
Jogeshwari ! Call Girls Service Mumbai - 450+ Call Girl Cash Payment 90042684...
Jogeshwari ! Call Girls Service Mumbai - 450+ Call Girl Cash Payment 90042684...Jogeshwari ! Call Girls Service Mumbai - 450+ Call Girl Cash Payment 90042684...
Jogeshwari ! Call Girls Service Mumbai - 450+ Call Girl Cash Payment 90042684...
 
Call Girls Mumbai Just Call 8250077686 Top Class Call Girl Service Available
Call Girls Mumbai Just Call 8250077686 Top Class Call Girl Service AvailableCall Girls Mumbai Just Call 8250077686 Top Class Call Girl Service Available
Call Girls Mumbai Just Call 8250077686 Top Class Call Girl Service Available
 
Coimbatore Call Girls in Thudiyalur : 7427069034 High Profile Model Escorts |...
Coimbatore Call Girls in Thudiyalur : 7427069034 High Profile Model Escorts |...Coimbatore Call Girls in Thudiyalur : 7427069034 High Profile Model Escorts |...
Coimbatore Call Girls in Thudiyalur : 7427069034 High Profile Model Escorts |...
 
8980367676 Call Girls In Ahmedabad Escort Service Available 24×7 In Ahmedabad
8980367676 Call Girls In Ahmedabad Escort Service Available 24×7 In Ahmedabad8980367676 Call Girls In Ahmedabad Escort Service Available 24×7 In Ahmedabad
8980367676 Call Girls In Ahmedabad Escort Service Available 24×7 In Ahmedabad
 
Call Girls Coimbatore Just Call 8250077686 Top Class Call Girl Service Available
Call Girls Coimbatore Just Call 8250077686 Top Class Call Girl Service AvailableCall Girls Coimbatore Just Call 8250077686 Top Class Call Girl Service Available
Call Girls Coimbatore Just Call 8250077686 Top Class Call Girl Service Available
 
Call Girls in Gagan Vihar (delhi) call me [🔝 9953056974 🔝] escort service 24X7
Call Girls in Gagan Vihar (delhi) call me [🔝  9953056974 🔝] escort service 24X7Call Girls in Gagan Vihar (delhi) call me [🔝  9953056974 🔝] escort service 24X7
Call Girls in Gagan Vihar (delhi) call me [🔝 9953056974 🔝] escort service 24X7
 
9630942363 Genuine Call Girls In Ahmedabad Gujarat Call Girls Service
9630942363 Genuine Call Girls In Ahmedabad Gujarat Call Girls Service9630942363 Genuine Call Girls In Ahmedabad Gujarat Call Girls Service
9630942363 Genuine Call Girls In Ahmedabad Gujarat Call Girls Service
 
Call Girls in Delhi Triveni Complex Escort Service(🔝))/WhatsApp 97111⇛47426
Call Girls in Delhi Triveni Complex Escort Service(🔝))/WhatsApp 97111⇛47426Call Girls in Delhi Triveni Complex Escort Service(🔝))/WhatsApp 97111⇛47426
Call Girls in Delhi Triveni Complex Escort Service(🔝))/WhatsApp 97111⇛47426
 

Drugs Used in Neoplastic Disorders

  • 2. 2 Safety precautions: chemotherapy drugs are hazardous For health professionals:  Many chemotherapy drugs are considered hazardous to healthy people. Reason why the nurses and doctors who give chemotherapy will take precautions to avoid direct contact with the drugs while administering them  Chemotherapy drugs can be dangerous to others in these ways: • They can cause abnormal changes in DNA. (They are mutagenic) • They may be able to alter development of a fetus or embryo, leading to birth defects (They are teratogenic) • They may be able to cause another type of cancer. (They are carcinogenic) • Some may cause skin irritation or damage. Nurses may wear special gloves, goggles, and gowns when preparing and giving you chemotherapy Pharmacists or nurses prepare the drugs in areas with special ventilation systems to avoid spattering and/or inhaling the droplets that can form while mixing Source: ACS Chemotherapy Principles
  • 3. Marc Imhotep Cray MD How to Learn Chemotherapeutic Agent 3 Begin by learning drug classes (antimetabolites, alkylating agents, antibiotic/anti-tumor agents, etc.) and their MOA  Classify each individual drug according to these groups  Learn toxicity of each individual ChemoTx agent . “Chemo man”  Step 1 likes testing on toxicities of chemotherapeutic drugs.  Briefly study drugs that can help neutralize toxic effects of ChemoTx agents. • Most important one to know is mesna, which can prevent hemorrhagic cystitis in patients receiving cyclophosphamide.
  • 4. Marc Imhotep Cray MD How to learn Chemotherapeutic Agent (2) 4  Learning clinical uses of chemotherapeutic agents should be last on our list.  You do not have to know which drugs are used for all types of cancers, but there are a few on which you should focus. • Know the drugs that are useful for testicular cancer (etoposide, bleomycin, and cisplatin), choriocarcinoma (methotrexate and vincristine/vinblastine), acute myelogenous leukemia (cytarabine), and brain tumors (nitrosoureas) • You should also know that 5-fluorouracil can be given topically for actinic keratosis
  • 5. Marc Imhotep Cray MD Important Terms to know 5  Cell cycle-nonspecific (CCNS) drug An anticancer agent that acts on tumor stem cells when they are traversing cell cycle and when they are in resting phase  Cell cycle-specific (CCS) drug An anticancer agent that acts selectively on tumor stem cells when they are traversing cell cycle and not when they are in G0 phase  Growth fraction The proportion of cells in a tumor population that are actively dividing  Myelosuppressant A drug that suppresses formation of mature blood cells such as erythrocytes, leukocytes, and platelets. This effect is also known as “bone marrow suppression”  Oncogene A mutant form of a normal gene that is found in naturally occurring tumors and which, when expressed in noncancerous cells, causes them to behave like cancer cells
  • 6. Marc Imhotep Cray MD Useful Abbreviations 6 ADC Antibody-drug conjugate Ara-C Cytarabine DHFR Dihydrofolate reductase FDA United States Food and Drug Administration FH 4 Tetrahydrofolate 5-FU 5-Fluorouracil Mesna Sodium 2-mercaptoethane sulfonate 6-MP 6-Mercaptopurine MTX Methotrexate 6-TG 6-Thioguanine
  • 7. Marc Imhotep Cray MD CHEMOTHERAPY I: OVERVIEW 7 1. The importance of tumor cell heterogeneity and the development of malignant cell resistance to chemotherapy as critical factors in determining treatment outcome. 2. The process of antineoplastic drug development. 3. The criteria determining the response to antineoplastic agents. 4. The toxicity of cancer chemotherapy. 5. The principles of management of the patient with cancer. 6. The importance of tumor staging in the management of the patient with cancer. 7. The principles of chemotherapy. 8. The rationale for the administration of adjuvant chemotherapy. See: ACS Chemotherapy Principles Doc. Core Concepts and Learning Objectives
  • 8. Marc Imhotep Cray MD Principles of Cancer Chemotherapy 8
  • 9. Marc Imhotep Cray MD Major Approaches to Treatment of Cancers 9 Tumor cells are shown in red and nontumor cells in green. *In clinical use. Others are experimental Brody Human Pharmacology: Mechanism-Based Therapeutics 6e. Elsevier, 2019. Pg. 1728.
  • 10. Marc Imhotep Cray MD Chemotherapy’s Main Clinical Usefulness Chemotherapy is useful for disseminated cancers that cannot be removed by surgery or as supplemental treatment after surgery or radiation  Using surgery or radiation to shrink tumor before ChemoTx ↑ number of dividing cells which ↑ effectiveness of ChemoTx  Most anticancer drugs affect cell division • They act preferentially on rapidly proliferating cells • Smaller tumors have a higher growth fraction o Consequently, they are more susceptible to anticancer drugs o Adjuvant ChemoTx is used w surgery or radiation to Tx undetectable metastases when they are small and highly sensitive to anticancer drugs • A greater proportion of nondividing cells will survive ChemoTx compared to dividing cells 10
  • 11. Marc Imhotep Cray MD Chemotherapy’s Objective 11 Objective of ChemoTx in any given individual patient may be:  Curative, to obtain complete remission (e.g., Hodgkin disease)  Palliative, to alleviate symptoms but with little expectation of complete remission (e.g., carcinoma of esophagus, with chemotherapy performed to ease the dysphagia)  Adjuvant, to improve chances for a cure or prolong period of disease-free survival when no detectable cancer is present but subclinical numbers of neoplastic cells are suspected (e.g., chemotherapy for breast cancer after surgical resection of all known tumor)  Debulking, to reduce tumor burden to allow for a more effective surgical removal of the tumor
  • 12. Marc Imhotep Cray MD Therapeutic Overview 12 Cancers in Which Complete Remission to Chemotherapy Is Common and Cures Are Achieved, Even in Advanced Disease*  Acute lymphocytic leukemia (adults and children)  Acute myelogenous leukemia  Hodgkin disease (lymphoma)  Non-Hodgkin lymphoma  Choriocarcinoma  Testicular cancer  Burkitt lymphoma  Ewing sarcoma  Wilms tumor  Small cell lung cancer  Ovarian cancer  Hairy cell leukemia *Depending on tumor type, complete remission may result in cure
  • 13. Marc Imhotep Cray MD Therapeutic Overview (2) 13 Cancers in Which Objective Responses Are Achieved, but Chemotherapy Often Does Not Have Curative Potential in Advanced Disease  Multiple myeloma  Breast cancer  Head and neck cancer  Colorectal carcinomas  Chronic lymphocytic leukemia  Chronic myelogenous leukemia  Transitional cell carcinoma of bladder  Gastric adenocarcinomas  Cervical carcinomas  Melanoma  Neuroblastoma  Non–small cell lung cancer
  • 14. Marc Imhotep Cray MD Therapeutic Overview (3) 14 Cancers in Which Only Occasional Objective Responses to Chemotherapy Are Achieved  Renal tumor  Pancreatic carcinomas  Hepatocellular carcinoma  Prostate carcinomas (hormone-unresponsive)
  • 15. 15 Basic Mechanisms by Which Antineoplastic Drugs Selectively Kill Tumor Cells  E represents enzymes, some of which are inhibited by these drugs  Key mechanisms of action for many of these drugs are: • Inhibition of DNA or RNA synthesis, • Production of miscoded nucleic acids, and • Formation of modified proteins Brody Human Pharmacology: Mechanism-Based Therapeutics 6e. Elsevier, 2019. Pg. 1748.
  • 16. 16 Cell Cycle Kinetics  Cancer cell population kinetics and cancer cell cycle are important determinants of actions and clinical uses of anticancer drugs • Some anticancer drugs exert their actions on cells undergoing cycling (cell cycle- specific [CCS] drugs), and • others (cell cycle-nonspecific [CCNS] drugs) kill tumor cells in both cycling and resting phases of cell cycle (although cycling cell are more sensitive)  CCS drugs are usually most effective when cells are in a specific phase of cell cycle  Both types of drugs are particularly effective when a large proportion of tumor cells are proliferating (ie, when growth fraction is high) Katzung BG, editor: Basic & Clinical Pharmacology, 12th ed. McGraw-Hill, 2012: Fig. 54–2
  • 17. Marc Imhotep Cray MD Cell Cycle Specific Anticancer Drugs 17  They are only effective against replicating cells, particularly malignancies w a high growth fraction  The cell cycle phases include: i. G1, phase after mitosis Some G1 cells can move into a resting, non-dividing state, G0 ii. S, DNA synthesis phase iii. G2, phase before mitosis iv. M, mitotic phase  Cell cycle phase specific drug classes include antimetabolites, bleomycin peptide antibiotics, vinca alkaloids (microtubule inhibitors), and etoposide i. Folic acid analog methotrexate (MTX) kills in S-phase (DNA synthesis phase) ii. Vincristine and vinblastine kill in M-phase (mitotic phase)
  • 18. Marc Imhotep Cray MD Cell Cycle Nonspecific Anticancer Drugs 18 Other drugs are cell cycle phase nonspecific  They are effective at killing nondividing cells as well as dividing cells  Cell cycle phase nonspecific drug classes include alkylating agents, most anticancer antibiotics, cisplatin, and nitrosoureas
  • 19. Marc Imhotep Cray MD The four phases of the cell cycle. G1 – the initial growth phase. S – the phase in which DNA is synthesized. G2 – the second growth phase in preparation for cell division. M – mitosis; where the cell divides to produce two daughter cells that continue the cell cycle. 19
  • 20. Marc Imhotep Cray MD Cancer therapy—cell cycle 20 First AID for the USMLE Step 1 2020, Pg. 438.
  • 21. Marc Imhotep Cray MD Cancer therapy—targets 21 First AID for the USMLE Step 1 2020, Pg. 438.
  • 22. Marc Imhotep Cray MD 22  Proportion of cancer cells that are actively proliferating will predict response to ChemoTx  Cells not actively proliferating are generally unresponsive to ChemoTx  As cancers increase in size, more cancer cells exit replicative pool  During course of ChemoTx , cells in replication are selectively killed, leaving behind cells out of pool that can continue on • If one were to completely surgically excise mass, then that would solve this issue, but even if mass cannot be removed, then at least one can debulk it, or use radiation therapy to kill more cells  ChemoTx protocols can be designed to include more agents, thereby reducing cancer cell resistance, or by giving cycles of ChemoTx that get more cancer cells into replicative pool CANCER CELLS ARE KILLED IN A FIRST-ORDER MANNER  a constant percentage (or proportion) of cancer cells are killed w each course of therapy  b/c of this log kill, additional rounds of chemotherapy are necessary in order to completely eradicate the tumor
  • 23. Marc Imhotep Cray MD The Log-Kill Hypothesis 23 Cytotoxic drugs act with first-order kinetics in a murine model of leukemia  In this model system a given dose kills a constant proportion of a cell population rather than a constant number of cells  The log-kill hypothesis proposes that magnitude of tumor cell kill by anticancer drugs is a logarithmic function For example, a 3-log-kill dose of an effective drug reduces a cancer cell population of 1012 cells to 109 (a total kill of 999 × 109 cells the same dose would reduce a starting population of 106 cells to 103 cells (a kill of 999 × 103 cells) • In both cases, dose reduces numbers of cells by 3 orders of magnitude, or “3 logs” • A key principle that stems from this finding and that is applicable to hematologic malignancies is an inverse relationship betw. tumor cell number and curability
  • 24. 24 Cancer Log-kill hypothesis Relationship, based on the log-kill hypothesis, of tumor cell number to 3 approaches to drug treatment and to no treatment (dashed line).  In the protocol diagrammed at top, infrequent treatment (indicated by arrows) prolongs survival but with recurrence of symptoms betw. Tx and eventual death.  With regimen diagrammed in middle section that is more intensive and begun earlier, cure results after many cycles of therapy.  In treatment diagrammed near bottom of graph, early surgery removes much of tumor burden, and intensive adjuvant chemotherapy has been used long enough to produce a cure. Katzung BG, editor: Basic & Clinical Pharmacology, 12th ed. McGraw-Hill, 2012: Fig. 54–1
  • 25. Marc Imhotep Cray MD Standard toxicities anticancer drugs 25 1. Myelosuppression is common because bone marrow is a rapidly proliferating tissue a. This is usually the dose-limiting side effect b. The leukopenia is greater than the thrombocytopenia, which is greater than the anemia c. Drugs for which bone marrow depression is not the dose-limiting toxicity include: i. Hormones ii. Vincristine iii. Bleomycin iv. Asparaginase v. Cisplatin vi. Monoclonal antibodies (MAbs)
  • 26. Marc Imhotep Cray MD Standard toxicities anticancer drugs (2) 26 2. Other rapidly proliferating cells that are affected include: a. GI epithelium b. Germinal epithelium c. Hair follicles
  • 27. Marc Imhotep Cray MD Standard toxicities anticancer drugs (3) 27 3. Nausea and vomiting are common side effects that can be managed with antiemetics, including: a. Phenothiazines such as prochlorperazine (Compazine) b. Cannabinoids such as dronabinol (Marinol) c. Dopamine receptor antagonists such as metoclopramide (Reglan) d. Ondansetron (Zofran), a 5-HT3 antagonist e. Glucocorticoids, such as dexamethasone f. Antihistamines, such as diphenhydramine (Benadryl) g. Benzodiazepines, such as lorazepam (Ativan)
  • 28. Marc Imhotep Cray MD Standard toxicities anticancer drugs (4) 28 4. Tissue necrosis may occur at site of injection 5. Some anticancer drugs have unique organ toxicities a. Anthracyclines (doxorubicin, daunorubicin, idarubicin, epirubicin and mitoxantrone) are cardiotoxic b. Bleomycin induces pulmonary fibrosis c. Vinca alkaloids (vincristine, vinblastine and vinorelbine); platinum compounds (cisplatin, carboplatin and oxaliplatin); and taxanes (paclitaxel and docetaxel) are neurotoxic d. Cisplatin, carboplatin, and methotrexate are nephrotoxic Anthracyclines (eg, Doxorubicin, Daunorubicin); prevent w Dexrazoxane
  • 29. Marc Imhotep Cray MD Minimizing ChemoTx Toxicities 29 6. Adverse side effects can be minimized by a variety of techniques including: a. Local perfusion of tumors b. Removing marrow pre-treatment and reimplanting it afterward c. Diuresis to prevent bladder toxicity d. Administering leucovorin (folinic acid) for megaloblastic anemia and prevention of MTX toxicity e. Urine alkalinization for MTX excretion f. Administering G-CSF (filgrastim) for neutropenia g. Administering allopurinol (Zyloprim) or rasburicase (Elitek) to treat hyperuricemia associated with tumor lysis syndrome, especially in leukemia and lymphoma patients
  • 30. 30 Common Cancer Chemotherapy Toxicities Drug Side Effect / Toxicity Mechanism Cyclophosphamide Hemorrhagic cystitis acrolein metabolite damages bladder epithelium (Rx: Mesna) 5-FU Diarrhea, photosensitivity damage of rapidly dividing GI epithelial cells interferes with fluid absorption from GI tract, resulting in watery bowel movements; 5-FU is “phototoxic” when exposed to sunlight Bleomycin Pulmonary Toxicity may be related to a combination of the production of toxic oxygen-free radicals & DNA damage in the lung Vincas Neuropathy, constipation Neuropathy may result from effects on neuronal microtubular assembly; constipation may in turn result from GI neuropathy Taxanes Neuropathy believed to result from disruption of neuronal microtubules Daunorubicin (Adriamycin) Cardiac toxicity evidence suggests that reactive oxygen species, generated by the interaction of doxorubicin with iron, damage cardiac myocytes, causing cellular damage & loss of muscle fibers, which can result in CHF Cisplatin Emesis, renal toxicity cisplatin is the most highly emetogenic agent; a proposed mechanism is that free radicals produced by cisplatin act on enterochromaffin cells to cause release of serotonin (5-HT); 5- HT3 receptor antagonists (e.g. ondansetron) are used to treat cisplatin induced emesis; renal toxicity may result from the ability of proximal tubular kidney cells to metabolize cisplatin to a Nephrotoxin, resulting in kidney damage Tamoxifen Uterine cancer, thrombosis Tamoxifen is a SERM and the increase in uterine (endometrial) cancer risk is related to tamoxifen's effect to behave as an estrogen partial agonist in the endometrium; the increase in deep vein thrombosis appears to result from an effect of tamoxifen to increase platelet aggregation by a poorly understood mechanism Alkylating agents Secondary leukemias DNA damage & resulting mutations caused by alkylating agents
  • 31. Marc Imhotep Cray MD Mesna (Mesnex) 31 Chemotherapy adjuvant  Mesna is used therapeutically to reduce incidence of hemorrhagic cystitis and hematuria when a patient receives ifosfamide or cyclophosphamide for cancer chemotherapy • These two anticancer agents, in vivo, may be converted to urotoxic metabolites, such as acrolein  MOA Mesna assists to detoxify these metabolites by reaction of its sulfhydryl group with α, β-unsaturated carbonyl containing compounds such as acrolein
  • 32. 32 Key Chemotoxicities “CHEMO MAN” First AID for the USMLE Step 1 2020, Pg. 444.
  • 33. Marc Imhotep Cray MD Other Problems with Chemotherapy 33 1. Immunocompromised patients usually have poorer responses to anticancer treatment 2. Centers of large tumors and CNS can serve as pharmacologic sanctuaries  Cancer cells are relatively protected from cytolytic effects of systemic chemotherapy 3. Some ChemoTx agents, particularly alkylating agents, can cause new, treatment-induced cancers up to several years after treatment  teratogenicity can also occur, again especially with alkylating agents
  • 34. Marc Imhotep Cray MD Other Problems with ChemoTx (2) 34 4. Resistance can develop a. Some cancers are inherently resistant to certain agents; other cancers can develop resistance by mutation, especially after long-term administration of low doses of drug b. Resistance is minimized by short-term, intensive, intermittent Tx w combinations of drugs c. Multidrug resistance occurs due to stepwise selection for permeability glycoprotein (P glycoprotein) i. P glycoprotein actively pumps drugs out of the cell ii. b/c P glycoprotein is a multidrug efflux pump, its activity provides cross-resistance for several structurally unrelated drug classes iii. Some organs naturally express high levels of P glycoprotein, including kidneys, intestines, liver, and pancreas Cells of these organs are therefore more resistant to chemotherapy
  • 35. Marc Imhotep Cray MD The Multidrug Resistance (MDR) Gene: Drug Specificity and Tissue Distribution 35 Drugs Affected by MDR Drugs Not Affected by MDR Adriamycin Methotrexate Daunomycin 6-Thioguanine Dactinomycin Cytarabine Plicamycin Cyclophosphamide Etoposide BCNU Vinblastine Bleomycin Vincristine Cisplatin Tissues with High MDR Expression Tissues with Low MDR Expression Colon Bone marrow Liver Breast Pancreas Ovary Kidney Skin Adrenal Central nervous system
  • 36. Marc Imhotep Cray MD Combination Chemotherapy 36 Combination Therapy is common and is often more effective against a wider variety of cell lines 1. Each drug in combination should be active against tumor to provide maximum cell killing within range of tolerance 2. Drugs should have different mechanisms of action to kill maximum number of cells in heterogeneous tumors 3. Drugs should have different toxicities so that they can all be given at full strength and emergence of resistance can be delayed 4. Drugs are usually admin. in Tx cycles and time must be allowed for host tissue recovery betw. cycles
  • 37. Marc Imhotep Cray MD Examples of Common Combination Drug Regimens 37 Terminology Cancer Drugs MOPP Hodgkin lymphoma Mechlorethamine, vincristine, procarbazine, prednisone ABVD Hodgkin lymphoma Doxorubicin, bleomycin, vinblastine, dacarbazine BEP Germ cell cancers Bleomycin, etoposide, cisplatin CMF Breast cancer Cyclophosphamide, methotrexate, 5-fluorouracil CAF Breast cancer Cyclophosphamide, doxorubicin, 5-fluorouracil CHOP CHOP-R Non-Hodgkin lymphoma Cyclophosphamide, doxorubicin, vincristine, prednisone Agents above + rituximab Hyper-CVAD Non-Hodgkin lymphoma Hyperfractionated cyclophosphamide, vincristine, adriamycin, dexamethasone FOLFIRI Cetuximab Colorectal cancer Leucovorin, 5-fluorouracil, irinotecan, cetuximab PVC Anaplastic oligodendrogliomas Procarbazine, vincristine, lomustine R-EPOCH B-cell non-Hodgkin lymphoma Rituximab, etoposide, prednisone, vincristine, cyclophosphamide, doxorubicin XELIRI Colorectal, esophageal, gastric cancer Capecitabine, irinotecan Brody Human Pharmacology: Mechanism-Based Therapeutics 6e. Elsevier, 2019. Pg. 1736.
  • 38. Marc Imhotep Cray MD Targeted therapies & Immunopharmacology 38 Targeted therapies are becoming increasingly important in the treatment of cancer  Examples include bevacizumab, which targets vascular endothelial growth factor;  I131 tositumomab, rituximab, and Y-90-ibritumomab tiuxetan, which target CD20 and are used for treatment of chemotherapy-refractory non- Hodgkin's lymphoma; and  Gefitinib and antibody cetuximab, which target epidermal growth factor receptor pathway  Immunotherapy includes biological response modifiers, which stimulate human immune system to destroy tumor cells  The α and β human interferons are examples of efficacious agents for hairy cell leukemia and certain skin cancers  Other compounds include tumor necrosis factor, human growth factors, and monoclonal antibodies
  • 39. Marc Imhotep Cray MD Targeted therapies & Immunopharmacology (2) 39 Notably important immuno-oncology targets are programmed death ligand 1 (PD-L1) and its cognate receptor, PD-1, which is found on activated T cells, B cells, and myeloid cells  PD-L1 up regulation allows melanoma, lung, renal, and gastric cancers to evade the host immune system PD-1 checkpoint inhibitors, such as nivolumab and pembrolizumab, have been approved Another example of effective immuno-oncology therapy is blocking antibody against cytotoxic T-lymphocyte–associated protein 4 (CTLA-4) Ipilimumab, which down regulates cytotoxic T-lymphocytes and is used for treatment of melanoma Learn more about Targeted Therapies & CA Immunopharmacology in Cancer Chemotherapeutic Drugs Notes_Pg. 28-40
  • 40. Marc Imhotep Cray MD 40 Chemotherapeutic Agents A Case-based Discussion Companion Notes: Cancer Chemotherapy Notes.pdf Cancer Chemotherapeutic Agents Rapid Review Q&A.pdf
  • 41. Marc Imhotep Cray MD 41 Lange Pharmacology Flashcards, Fourth Edition, M-H 2018. Pg.264.
  • 42. Marc Imhotep Cray MD Classification of Anti-Neoplastic Agents by Site of Action 42
  • 43. Marc Imhotep Cray MD Classification of Anti-Neoplastic Agents by Site of Action 43 (See Reproductive)
  • 44. Marc Imhotep Cray MD CHEMOTHERAPY II: ALKYLATING AGENTS 44 1. The characteristic indications for the alkylating agents and plant alkaloids commonly used in chemotherapy. 2 .The mechanism of action and mechanisms of resistance for the alkylating agents and plant alkaloids commonly used in chemotherapy. 3. The principal adverse effects of the commonly used alkylating agents and plant alkaloids. Relevant Drugs: Cyclophosphamide Ifosfamide Cisplatin Carboplatin Oxaliplatin Busulfan Nitrosureas Procarbazine Core Concepts and Learning Objectives
  • 45. Marc Imhotep Cray MD Case#1 45 A 72-year-old woman presents to the emergency room complaining of the onset of pain and bleeding with urination over the past 24 hours. She denies any fever, flank pain, or history of kidney stones. Her past medical history is significant for breast cancer, for which she is currently receiving chemotherapy. Urinalysis is significant for copious amounts of blood and red blood cells. You admit the patient to the hospital for a urology workup and you begin to wonder if her symptoms might be related to her chemotherapeutic regimen. What is the Medication?
  • 46. Marc Imhotep Cray MD Cyclophosphamide 46 Similar Drugs Ifosfamide Mechanism of Action  Cyclophosphamide is metabolized to a hydroxylated intermediate compound by cytochrome P-450 system in liver This hydroxylated intermediate then acts as an alkylating agent and cross-links DNA, thereby decreasing DNA and RNA synthesis  drug has also been implicated in the suppression of B- and T-cell function Clinical Uses  Used in treatment of a variety of solid and hematologic malignancies (e.g., breast, ovarian, non-Hodgkin lymphoma)  Immunosuppressive agent used to treat rheumatoid arthritis, SLE, Wegener granulomatosis, and nephrotic syndrome
  • 47. Marc Imhotep Cray MD Cyclophosphamide (2) 47 Side Effects/AEs Hemorrhagic cystitis (caused by accumulation of the metabolite, acrolein, in urine; incidence can be greatly reduced by giving patient copious fluids as well as MESNA [sodium 2-mercaptoethane sulfonate]); bone marrow suppression; GI upset; alopecia; infertility Other Mechlorethamine is another alkylating agent that is used as part of MOPP protocol (mechlorethamine, vincristine, procarbazine, and prednisone) in Tx of Hodgkin’s lymphoma. Procarbazine is another alkylating agent that is also used as part of MOPP protocol in treatment of Hodgkin’s disease as well as some brain tumors.  A metabolite of procarbazine inhibits monoamine oxidase, and thus patients taking this drug should not ingest foods containing tyramine (wine, smoked meat, cheese) due to risk of norepinephrine buildup causing a hypertensive crisis
  • 48. Marc Imhotep Cray MD Case #2 48 A 51-year-old man presents to your oncology clinic for evaluation of his CML. He will be undergoing bone marrow transplantation in the coming weeks, and he will need to have his marrow ablated prior to his transplantation. You inform him that his bone marrow will be ablated with a combination of medications, including an alkylating agent that acts to damage DNA via the cross-linking of DNA strands. You explain to him that this medication has many side effects, including adrenal insufficiency, seizures, and pulmonary fibrosis. The patient realizes that there may be serious side effects, but is more than willing to undergo the treatment to prepare for potentially life- saving bone marrow transplantation. What is the Medication?
  • 49. Marc Imhotep Cray MD Busulfan 49 Mechanism of Action Busulfan acts as an alkylating agent that acts to damage DNA via cross-linking of DNA strands Clinical Uses  Treatment of CML  Also used in combination with other drugs to ablate bone marrow before bone marrow transplantation Side Effects/AEs Pulmonary fibrosis; bone marrow suppression; adrenal insufficiency with associated hyperpigmentation; seizures Black Box Warning: Busulfan is a potent cytotoxic drug that causes PROFOUND MYELOSUPPRESSION at the recommended dosage. It should be administered under the supervision of a qualified physician who is experienced in allogeneic hematopoietic stem cell transplantation, the use of cancer chemotherapeutic drugs and the management of patients with severe pancytopenia.
  • 50. Marc Imhotep Cray MD Case#3 50 A 42-year-old man presents to your neurology clinic complaining of severe headaches that have been present almost every day for the past month. The pain is usually lateralized to the right side and is often present upon waking. Over the past few days, he has also experienced several episodes of projectile vomiting. Physical examination is significant for the presence of papilledema on funduscopic examination. You become concerned and send the patient for an MRI of the brain. The imaging study reveals a mass in the right temporal lobe. Characteristics of the mass on imaging are consistent with a likely malignancy. You refer the patient to a neurosurgeon as well as to an oncologist. You believe that he will need chemotherapy and radiation to treat his condition and you suspect that his chemotherapy regimen will likely include a class of drugs that can cross the blood-brain barrier and then cross-link DNA strands to inhibit DNA synthesis. What is the Medication?
  • 51. Marc Imhotep Cray MD Nitrosoureas 51 Similar Drugs Nitrosoureas include carmustine, lomustine, semustine, and streptozocin Mechanism of Action Drugs in this class are alkylating agents, which act to cross-link DNA strands, and thereby result in the inhibition of DNA and RNA synthesis Clinical Uses  Chemotherapeutic agents used in the treatment of brain tumors (e.g., glioblastoma multiforme), multiple myeloma, and lymphomas.  Streptozocin is also used in the treatment of insulinomas Side Effects/AEs CNS toxicity; bone marrow suppression; nephrotoxicity; pulmonary fibrosis.
  • 52. Marc Imhotep Cray MD Nitrosoureas (2) 52 Black Box Warnings: BONE MARROW SUPPRESSION - notably thrombocytopenia & leukopenia. CARMUSTINE PULMONARY TOXICITY is dose related. Risk increases with cumulative doses > 1400 mg/m2, history of lung disease & duration of therapy. Delayed cases of pulmonary fibrosis have been reported 15 years after administration. Of Note: Nitrosoureas are one of the few chemotherapeutic agents that are highly lipid-soluble and cross the blood-brain barrier, and thus are useful in treating CNS malignancies
  • 53. Marc Imhotep Cray MD Case#4 53 A 76-year-old man presents to your clinic complaining of a ringing in both of his ears that has been present over the past month. He also tells you that he seems to be having more trouble hearing during this period as well. His past medical history is significant for bladder cancer, for which he is currently being treated with chemotherapy. Physical examination is remarkable for gross sensorineural hearing loss bilaterally. You suspect that his hearing loss is related to one of his medications and you decide to call his oncologist to see if there is an alternative chemotherapeutic agent that can be used to treat his malignancy, but will not cause acoustic nerve damage. What is the Medication?
  • 54. Marc Imhotep Cray MD Cisplatin (cis–diamminedichloroplatinum) 54 Similar Drugs Carboplatin Mechanism of Action Cisplatin acts as an alkylating agent, thereby inhibiting DNA and RNA synthesis by binding DNA strands and leading to formation of cross-links between strands Clinical Uses Chemotherapeutic agent used in treatment of many genitourinary tumors (testicular, ovarian, bladder), other solid tumors (small cell lung cancer, stomach, esophageal) and lymphomas Side Effects/AEs Nephrotoxicity; ototoxicity; neurotoxicity; bone marrow suppression
  • 55. Marc Imhotep Cray MD Cisplatin(2) 55 Black Box Warnings: NEPHROTOXICITY, MYLEOSUPPRESSION & OTOTOXICITY 1.NEPHROTOXICITY is cumulative, dose related and severe. It has been noted in 28-36% of patients treated with a single dose of 50 mg/m2. It is first noted during the 2nd week after a dose & is manifested by elevations in BUN, creatinine & serum uric acid. Renal function must return to normal before another dose of cisplatin can be given. 2.MYLEOSUPPRESSION: dose related and severe. Occurs in 25-30% of patients. Nadirs in circulating platelets & leukocytes occur within 18-23 days in most patients, with recovering by day 39. 3.Nausea & vomiting - dose related and severe. Seen in almost all patients & begins within 1-4 hrs after treatment & may last for 24 hrs or more (may persist up to 1 week). 4.OTOTOXICITY is significant (observed in up to 31% of patients treated with a single dose of 50 mg/m2) - may be more pronounced in children 5.Anaphylactic like reactions have been reported (epinephrine, corticosteroids & antihistamines have been effective in alleviating symptoms). 6.Avoid overdose or confusion with carboplatin
  • 56. Marc Imhotep Cray MD Cisplatin(3) 56 Important Notes: Cisplatin-induced nephrotoxicity is dose-dependent and can be mitigated with use of amifostine acts as a scavenger of free radicals in tissues treated w cisplatin, as well as aggressive IV fluids to induce diuresis Mitomycin is another alkylating agent that damages DNA via cross- linking of DNA strands  It is mostly used in combination with other agents to treat certain solid malignancies (e.g., breast, gastric, esophageal, pancreas, bladder)
  • 57. Marc Imhotep Cray MD CHEMOTHERAPY III: ANTI-METABOLITES 57 1. The characteristic indications for the use on antimetabolites commonly used in the treatment of cancer. 2. The mechanism of action of the antimetabolites commonly used in the treatment of cancer. 3. The mechanism of resistance to antimetabolites 4. The major adverse effects associated with antimetabolites commonly used in the treatment of cancer. Relevant Drugs Methotrexate Pemetrexed Cytarabine Gemcitabine 5-Fluorouracil Capecitabine 6-Mercaptopurine 6-Thioguanine Core Concepts and Learning Objectives
  • 58. Marc Imhotep Cray MD Case#5 58 A 42-year-old woman presents to your rheumatology clinic for follow-up of her rheumatoid arthritis. She is currently being treated with nonsteroidal anti-inflammatory drugs (NSAIDs) and steroids, but she states that her symptoms are still severe. Physical examination demonstrates swelling of the proximal interphalangeal and metacarpophalangeal joints of both hands, ulnar deviation of the fingers, and subcutaneous nodules over both of her elbows. You decide to start the patient on a medication that acts to inhibit dihydrofolate reductase and will thereby decrease her body’s supply of activated folic acid. What is the Medication?
  • 59. Marc Imhotep Cray MD Methotrexate 59 Mechanism of Action  MTX inhibits dihydrofolate reductase, enzyme that transforms folic acid to active form, FH4 FH4 is a necessary precursor for formation of thymidylate  Without FH4, thymidylate supplies are diminished, thereby resulting in decreased DNA synthesis  MTX is a S phase specific ChemoTx agent Clinical Uses  Chemotherapeutic agent used to treat a variety of malignancies (e.g., ALL, lymphomas, breast cancer, head and neck cancer, osteosarcoma, choriocarcinoma).  Immunosuppressant used to treat many autoimmune disorders (e.g., RA, Crohn’s disease, scleroderma, psoriasis)  Abortifacient when administered with a prostaglandin (used to treat missed abortions or small ectopic pregnancies) Side Effects/AEs Bone marrow suppression; nephrotoxicity (may crystallize in renal tubules); hepatotoxicity (steatosis); teratogen; pulmonary toxicity; mucositis; seizures; GI upset; alopecia
  • 60. Marc Imhotep Cray MD MTX (2) 60 Important Notes:  Leucovorin (folinic acid) is often given in conjunction w MTX in order to minimize bone marrow suppression, which occurs b/c of lack of folate coenzymes  In the cell, leucovorin is converted to N5N10- methylene-FH4, which is a downstream product of reaction catalyzed by dihydrofolate reductase • Therefore, leucovorin acts to bypass the inhibited enzyme and provides an adequate supply of activated folate  MTX has same molecular target as antibiotic, trimethoprim, and anti-parasitic drug, pyrithiamine, but MTX work to target different isoforms of dihydrofolate reductase
  • 61. Marc Imhotep Cray MD 61 Black Box Warnings (abbreviated version): 1.Because of the possibility of serious toxic (and potentially fatal) reactions, methotrexate should be used only in life threatening neoplastic diseases or severe recalcitrant psoriasis or rheumatoid arthritis which is not responsive to other forms of therapy. 2.Include the use of antimetabolite (Leucovorin) therapy 3.Patients should be closely monitored for bone marrow, liver, kidney & lung toxicity. 4.Methotrexate has been reported to cause fetal death & congenital abnormalities. Not recommended for women of child bearing potential unless the benefits outweigh the risks. 5.Unexpected bone marrow suppression, aplastic anemia & GI toxicity have been reported with high doses of methotrexate & concurrent use of NSAIDs. 6.Methotrexate causes liver toxicity, fibrosis & cirrhosis, but generally only after prolonged use. 7.Malignant lymphomas, which may regress following withdrawal of methotrexate may occur with low dose treatment. 8.Diarrhea & ulcerative stomatitis require interruption of therapy, otherwise hemorrhagic enteritis & death from intestinal perforation may occur. 9.Similar to other cytotoxic drugs, methotrexate cay cause a “tumor lysis syndrome” in patients with rapidly growing tumors. Appropriate measures should be taken should this occur. 10.Severe & occasionally fatal skin reactions have been reported. 11.Potentially fatal opportunistic infections (e.g. Pneumocystis jiroveci pneumonia) may occur. 12.Methotrexate combined with radiotherapy may increase the risk for necrosis of soft tissues & osteonecrosis.
  • 62. Marc Imhotep Cray MD Case#6 62 A 10-year-old girl presents to your oncology clinic for follow-up of her acute lymphoblastic leukemia, for which she is currently receiving chemotherapy. She was recently discharged from the hospital 2 days prior to her appointment at your clinic. Her mother tells you that the patient developed an episode of acute gout while in the hospital, after which she was placed on allopurinol to prevent future gouty attacks. You immediately decide to alter the doses of the patient’s chemotherapeutic regimen, as allopurinol indirectly inhibits the metabolism of one of the drugs in the patient’s chemotherapeutic regimen and can thus result in increased serum levels of this drug. What is the Medication?
  • 63. Marc Imhotep Cray MD 6-Mercaptopurine 63 Similar Drugs Azathioprine Mechanism of Action 6-MP is converted into thio-IMP (6-Thioinosine 5'-monophosphate) by HGPRT (Hypoxanthine-guanine phosphoribosyltransferase), an enzyme involved in the salvage pathway. Thio-IMP acts as a purine analogue, which inhibits purine synthesis through feedback inhibition of several enzymes involved in de novo purine synthesis. Thio-IMP is also converted to thio-GTP, which is incorporated into DNA and RNA, leading to strand instability Clinical Uses Chemotherapeutic agent used primarily in treatment of leukemias (especially acute lymphoblastic leukemia) as well as non-Hodgkin lymphoma Immunosuppressant used to treat inflammatory bowel disease and psoriasis and prevent allograft rejection Side Effects/AEs Bone marrow suppression; GI upset; increased susceptibility to infection
  • 64. Marc Imhotep Cray MD 6-MP 64 Improtant Notes: DDI: 6-MP is metabolized by xanthine oxidase, an enzyme involved in purine degradation Allopurinol, a drug used to treat hyperuricemia and gout, is a potent inhibitor of xanthine oxidase  When allopurinol and 6-MP are given simultaneously levels of serum 6- MP become ↑ b/c of ↓ metabolism of 6-MP by xanthine oxidase Azathioprine a structural analogue of 6-MP & isconverted to 6-MP within cell  It is primarily used as an immunosuppressant in autoimmune diseases such as SLE, IBD, RA, ITP, multiple sclerosis, and allograft rejection  Side effects are similar to those caused by 6-MP, and include bone marrow toxicity as well as GI upset  Both azathioprine and 6-MP are S-phase specific chemotherapeutic agents
  • 65. Marc Imhotep Cray MD Case #7 65 A 58-year-old man presents to your office for a follow-up visit. He was recently diagnosed with colon cancer and underwent colonic resection. He is currently receiving chemotherapy as an adjuvant treatment. At this visit, he tells you that he has noticed increased sensitivity of his eyes to light as well as painful ulcers on his lips and in his mouth. After examining his eyes and oral pharynx, you tell him that you suspect that his photosensitivity and mucosal ulcers are likely a side effect of one of the chemotherapeutic drugs being used to treat his colon cancer. What is the Medication?
  • 66. Marc Imhotep Cray MD 5-Fluorouracil 66 Mechanism of Action 5-FU is converted into 5-FdUMP, which then acts to inhibit thymidylate synthase, enzyme responsible for synthesizing thymine nucleotides Inhibition of thymidylate synthase leads to a disruption of nucleotide synthesis b/c of a lack of thymidine; 5-FU can also be incorporated into RNA, leading to dysfunctional RNA processing Clinical Uses Chemotherapeutic agent used to treat a variety of adenocarcinomas (gastric, pancreatic, colon, breast, ovarian) and for basal cell carcinoma Side Effects/AEs Bone marrow suppression; photosensitivity; anorexia; oral ulcerations. Of Note 5-FU is a cell cycle-specific drug, acting during S phase of cell cycle
  • 67. Marc Imhotep Cray MD Case#8 67 A 36-year-old woman presents to your oncology clinic for follow-up of her acute myelogenous leukemia. She has been receiving several chemotherapeutic agents for the treatment of her disease. Her most recent treatment involved a cell cycle-specific drug that acts by competitively inhibiting DNA polymerase. She tells you that lately she has been feeling extremely tired and that she had a nosebleed last week that took a long time to stop bleeding. On physical examination, you note that she is a pale, thin woman with a 3/6 systolic flow murmur at the left upper sternal border and a diffuse petechial rash over her limbs. Concerned, you send her to the emergency room for blood tests, which you suspect will reveal pancytopenia that is most likely caused by her recent chemotherapy treatment. What is the Medication?
  • 68. Marc Imhotep Cray MD Cytarabine (cytosine arabinoside) 68 Mechanism of Action In the cell, cytarabine is converted to araCTP (arabinofuranosylcytosine triphosphate), which acts to competitively inhibit DNA polymerase and thus impair DNA synthesis. araCTP can also be incorporated into DNA, leading to termination of DNA strand elongation during DNA synthesis  Cytarabine is a cell cycle-specific agent, acting during S phase of cell cycle Clinical Uses Chemotherapeutic agent for the treatment of AML as well as lymphomas Side Effects/AEs Bone marrow suppression with resulting pancytopenia; alopecia; GI upset; ataxia Black Box Warnings: The main toxic effect of cytarabine injection is BONE MARROW SUPPRESSION with leukopenia, thrombocytopenia, and anemia.
  • 69. Marc Imhotep Cray MD Cladrabine 69 Cladrabine acts by interfering with DNA synthesis as well; After becoming activated through phosphorylation, cladrabine incorporates into growing DNA strands and leads to DNA strand breakage  Because activated form of drug also inhibits ribonucleotide reductase, it also induces a depletion of adenosine triphosphate (ATP)  Cladrabine is a cell cycle-non-specific agent, and thus can induce cell death in resting as well as proliferating cancer cells USE It is used primarily for treatment of hairy cell leukemia Side effects/AEs include bone marrow suppression, neurotoxicity, and nephrotoxicity.
  • 70. Marc Imhotep Cray MD CHEMOTHERAPY V: NATURAL PRODUCTS: PLANT ALKALOIDS 70 Vinca alkaloids: Vincristine, Vinblastine and Vinorelbine. Taxanes: Paclitaxel and Docetaxel Podophyllotoxins: Etoposide and Tenisopide Camptothecan analogs: Irinotecan and Topotecan
  • 71. Marc Imhotep Cray MD Case#9 71 A 67-year-old man presents to your clinic complaining of a 20-pound weight loss, night sweats, and worsening malaise. He states that he has multiple family members who have been diagnosed with cancer, but he is not sure which type they have. He also says that he has not had any cancer screening, including colonoscopies, because of “insurance issues.” A stool occult blood test is positive, and a subsequent colonoscopy shows cancerous growths in the sigmoid colon. A follow-up PET scan shows metastatic spread of his cancer. After delivering the diagnosis to the patient and his family, you begin to discuss initial treatment options. You note that one medication used to treat metastatic colon cancer works by inhibiting the enzyme topoisomerase I. What is the Medication?
  • 72. Marc Imhotep Cray MD Irinotecan 72 Similar Drugs Topotecan Mechanism of Action Irinotecan acts by inhibiting topoisomerase I, which leads to excessive DNA supercoiling and eventual DNA strand breakage and cell death Clinical Uses  Chemotherapeutic agent used in Tx of recurrent or metastatic solid tumors (notably metastatic colon cancer for irinotecan and metastatic ovarian or cervical cancer for topotecan)  Other cancers treated with these agents include pancreatic, esophageal, and small-cell lung cancer Side Effects/AEs Bone marrow suppression; GI upset. Notes: Irinotecan and topotecan are cell cycle-specific drugs, active during the S phase of cell cycle
  • 73. Marc Imhotep Cray MD Irinotecan(2) 73 Black Box Warnings: SEVERE DIARRHEA Both early & late (after >24 hrs) forms of DIARRHEA may occur & can be SEVERE. Early diarrhea may be accompanied by cholinergic symptoms & can be reduced by atropine. Late diarrhea can be life-threatening due to dehydration, electrolyte imbalance or sepsis. Late diarrhea should be treated promptly with loperamide. Patients with diarrhea should be carefully monitored & given fluid, electrolytes, antibiotics if needed. Administration of Irinotecan should be discontinued if severe diarrhea occurs.
  • 74. Marc Imhotep Cray MD Case#10 74 A 21-year-old man presents to the emergency room complaining of unsteady gait, which he began to notice over the past week. Upon further questioning, you learn that he was recently diagnosed with Hodgkin lymphoma and is currently receiving chemotherapy for his disease. Physical examination is significant for hyporeflexia in his lower extremities and right foot-drop. Laboratory studies are unremarkable and, notably, his platelets, white blood cell counts, and red blood cell counts are normal. As you prepare to admit the patient to the neurology service, you wonder if his neurologic deficits are related to his chemotherapeutic regimen. What is the Medication?
  • 75. Marc Imhotep Cray MD Vincristine and Vinblastine 75 Mechanism of Action Vincristine (VC) and Vinblastine (VB) bind to tubulin, a microtubular protein, thereby leading to the depolymerization of mitotic spindle. Thus, cells are unable to progress past metaphase during mitosis or meiosis, leading to decreased cellular proliferation Clinical Uses  VC: Chemotherapeutic agent used in the treatment of ALL, lymphomas (part of the MOPP protocol used to treat Hodgkin’s disease), Wilms’ tumor, and Ewing’s sarcoma.  VB: Chemotherapeutic agent used to treat testicular cancer, breast cancer, non-small cell lung cancer, and various lymphomas Side Effects/AEs  VC: Peripheral neuropathy; GI upset; alopecia; myelosuppression is rare and usually mild when it does occur  VB: Bone marrow suppression; GI upset; alopecia Note: VC and VB are cell cycle-specific drugs, acting during M phase of cell cycle
  • 76. Marc Imhotep Cray MD Case#11 76 A 43-year-old woman with metastatic breast cancer returns to your oncology clinic. She has undergone combination chemotherapy for her metastatic disease without a positive response. She is quite distraught and asks if her chemotherapy can be changed. You inform her that there is a new therapy for treatment of breast cancer in case combination chemotherapy fails. You decide to switch her regimen to include this other medication, which acts by interfering with the ability of the mitotic spindle to break down and thus halting mitosis. What is the Medication?
  • 77. Marc Imhotep Cray MD Paclitaxel 77 Similar Drugs Docetaxel Mechanism of Action Paclitaxel binds to tubulin, a microtubular protein. The paclitaxel-tubulin complex acts to promote stabilization and polymerization of mitotic spindle leads to halt of mitosis during metaphase (anaphase cannot occur) Clinical Uses  Chemotherapeutic agent used in Tx of ovarian and breast cancer, small cell cancer of lung, advanced Kaposi sarcoma, and head and neck cancers  Paclitaxel has been used in past to coat coronary artery stents so as to prevent restenosis of artery at site of stent Side Effects/AEs Hypersensitivity reactions (rash); bone marrow suppression; neurotoxicity
  • 78. Marc Imhotep Cray MD Paclitaxel (2) 78 Black Box Warnings: ANAPHYLAXIS & MYLEOSUPPRESSION 1.ANAPHYLAXIS & severe hypersensitivity reactions with dyspnea, hypotension, urticaria & angioedema requiring treatment have occurred in 2-4% of patients. Fatal reactions have occurred despite premedication. 2.ALL PATIENTS SHOULD BE PRETREATED with CORTICOSTEROIDS, DIPHENHYDRAMINE & H2 ANTAGONISTS. 3.Paclitaxel should not be given to patients with solid tumors that have baseline neutrophil counts of less than 1500 cells/mm3 & should NOT be given to patients with AIDS-related Kaposi's sarcoma if their baseline neutrophil count is less than 1000 cells/mm3. 4.Frequent peripheral blood cell counts should be performed to monitor the occurrence of bone marrow suppression (primarily neutropenia).
  • 79. Marc Imhotep Cray MD CHEMOTHERAPY IV: ANTI-TUMOR ANTIBIOTIC AGENTS 79 1. The characteristic indications, mechanism of action, mechanism of resistance and adverse effects of doxorubicin and the other commonly used antibiotic and anti-tumor agents. 2. The concept of multiple drug resistance and its effects on effective chemotherapy. 3. The concept of cumulative toxicity and schedule independent toxicity. 4. The concept of the hormonally-sensitive neoplasms and their treatment. 5. The major side effects associated with antibiotic and anti- tumor agents commonly used in chemotherapy. Relevant Drugs: Doxorubicin Daunorubicin Idarubicin Dactinomycin Bleomycin Core Concepts and Learning Objectives
  • 80. Marc Imhotep Cray MD Case#13 80 A 14-year-old boy presents to your office for initiation of chemotherapy to treat his acute lymphoblastic leukemia. You explain to the boy and his mother that you will be using several different chemotherapy agents to treat his disease. One of the agents you will be administering acts by blocking DNA and RNA synthesis; however, this drug also causes the production of oxygen free radicals, which can damage cardiac tissue when given at high doses. You assure the patient that you will be carefully monitoring levels of this drug so as to avoid cardiac toxicity if possible. What is the Medication?
  • 81. Marc Imhotep Cray MD Anthracycline Antibiotics 81 Class of anthracyclines includes doxorubicin, daunorubicin, and idarubicin Mechanism of Action Anthracyclines block DNA and RNA synthesis both through direct steric inhibition as well as by inhibiting topoisomerase II. They also cause production of oxygen free radicals, which lead to membrane damage Clinical Uses  Doxorubicin: used in treating a variety of solid tumors (breast, ovary, bladder, endometrium, stomach, lung) as well as in treating a variety of hematologic malignancies (acute leukemia, lymphoma, multiple myeloma) and in treating AIDS-related Kaposi sarcoma.  Daunorubicin: used in treating acute leukemia (AML, ALL, CML) and neuroblastoma  Idarubicin: used in treating acute myeloid leukemia
  • 82. Marc Imhotep Cray MD Anthracyclines(2) 82 Side Effects/AEs Cardiac toxicity; bone marrow suppression; GI distress; alopecia Important Notes:  The cardiotoxic effects of anthracyclines are caused, in part, by production of oxygen free radicals within myocardial cells and subsequent damage of myocyte.  This adverse effect is cumulative, dose-dependent, and can be largely avoided by keeping doses below a certain lifetime dose and by administering dexrazoxane a compound that acts to decrease free radical formation
  • 83. Marc Imhotep Cray MD Anthracyclines (3) doxorubicin, daunorubicin 83 Black Box Warnings: SEVERE MYELOSUPPRESSION & MYOCARDIAL TOXICITY 1.Daunorubicin for injection must be given into a rapidly flowing i.v. infusion. It must never be given by i.m. or s.c. routes due to local tissue necrosis. 2.MYOCARDIAL TOXICITY manifested in its most severe form as potentially fatal CHF may occur months to years after termination of therapy. The incidence increases after cumulative doses exceed 400-550 mg/m2 in adults or 300 mg/m2 in children >2yo, or 10 mg/kg in children <2yo. 3.SEVERE MYELOSUPPRESSION occurs at therapeutic doses, which may lead to infection or hemorrhage. 4.dosage should be reduced in patients with impaired hepatic or renal function. 5.Daunorubicin should only be given in a facility where physicians & staff are capable of rapidly responding to severe hemorrhagic conditions and/or overwhelming infection.
  • 84. Marc Imhotep Cray MD Case#14 84 A 2-year-old boy is brought to your oncology clinic after having been recently diagnosed with Wilms tumor. Physical examination is significant for a palpable left flank mass. You explain to the child’s mother that the best course of treatment will include surgical excision of the tumor and chemotherapy. You recommend the use of at least two drugs, one of which acts by interfering with RNA synthesis through the inhibition of DNA dependent RNA polymerase. Since this drug can also cause bone marrow suppression, the child’s blood cell counts, especially his platelets and leukocytes, will have to be monitored closely. What is the Medication?
  • 85. Marc Imhotep Cray MD Dactinomycin 85 Similar Drugs Plicamycin Mechanism of Action Dactinomycin intercalates betw. cytosine-guanine base pairs of DNA, thereby acting to inhibit DNA-dependent RNA polymerase, and thereby impairing both DNA and RNA synthesis. Clinical Uses Used for several solid tumors including Wilms tumor, Ewing sarcoma, and metastatic testicular cancer. Also used in conjunction w MTX to treat gestational choriocarcinoma Side Effects/AEs Bone marrow suppression; GI upset; skin abnormalities at sites that have previously received radiation (“radiation recall”) Note Plicamycin has a similar MOA as dactinomycin, but is primarily used for Tx of some testicular cancers and Paget’s disease of bone
  • 86. Marc Imhotep Cray MD Case#15 86 A 69-year-old man presents to your clinic complaining of shortness of breath that has become progressively worse over the past month. He tells you that he was diagnosed with testicular cancer 4 months ago, for which he is currently receiving chemotherapy. Physical examination is significant for an oxygen saturation of 89% on room air and decreased air movement on lung examination. When imaging studies reveal pulmonary fibrosis, you begin to suspect that this patient’s symptoms are likely related to his chemotherapeutic regimen. What is the Medication?
  • 87. Marc Imhotep Cray MD Bleomycin 87 Mechanism of Action Bleomycin binds to DNA and triggers the formation of oxygen free radicals. The oxygen radicals then act to damage the DNA, leading to strand breaks and inhibition of DNA synthesis. Clinical Uses Chemotherapeutic agent for the treatment of testicular tumors, squamous cell carcinomas (head and neck, skin, penis, cervix), and Hodgkin’s lymphoma. Side Effects/AEs Pulmonary fibrosis; allergic reactions (fever, anaphylaxis); mucositis; skin changes Note that myelosuppression is rare with bleomycin Of Note Bleomycin is cell cycle-specific, thereby leading to accumulation of cells in G2 phase of cell cycle
  • 88. Marc Imhotep Cray MD Bleomycin (2) 88 Black Box Warnings: PULMONARY FIBROSIS is associated with bleomycin treatment. The most common presentation is pneumonitis, progressing to pulmonary fibrosis. It is more common in elderly patients receiving more than 400 units total dose, but has been seen in younger patients & those treated with lower doses. A severe IDIOSYNCRATIC REACTION of hypotension, mental confusion, chills, fever & wheezing has also been reported in 1% of lymphoma patients undergoing bleomycin treatment.
  • 89. Marc Imhotep Cray MD CHEMOTHERAPY V: TARGETED THERAPIES OF CANCER MISCELLANEOUS AGENTS 89 1. The concept of “targeted cancer therapy”. 2. The role of tyrosine kinases as targets for cancer therapy. 3. The potential benefits and toxicities of the commercially available targeted therapies. 4. The uses and side effects of miscellaneous anti- cancer agents. Relevant Drugs Imatinib mesylate (Gleevec) Erlotinib (Tarceva) Cetuximab (Erbitux) Trastuzumab (Herceptin) Bevacizumab (Avastin) Rituximab (Rituxin) Asparaginase Hydroxyurea All-trans-retinoic acid Arsenic trioxide
  • 90. Marc Imhotep Cray MD Case#16 90 A 56-year-old gentleman with a past medical history significant for hypertension and tobacco use presents to your office complaining of hematuria. On physical exam, you palpate a flank mass on the left. Ultrasound confirms an irregular, non- cystic, vascular lesion in his left kidney. Follow-up biopsy confirms your suspicion that the patient has a malignant growth in his kidney. You suspect that the patient’s chemotherapy regimen will include an agent to inhibit angiogenesis. What is the Medication?
  • 91. Marc Imhotep Cray MD Bevacizumab 91 Mechanism of Action Bevacizumab is a monoclonal antibody specific for vascular endothelial growth factor (VEGF). By blocking the activity of VEGF, bevacizumab prevents angiogenesis, reducing vascular supply to tumors and retarding their growth. Clinical Uses  Chemotherapeutic agent used in Tx of solid tumors (ovarian, colorectal, cervical, renal cell carcinoma)  It is also used to treat dysregulated angiogenesis in wet age related macular degeneration
  • 92. Marc Imhotep Cray MD Bevacizumab (2) 92 Side Effects/AEs Arterial thromboembolism; hypertension; hemorrhage. Related Notes  Cetuximab is a monoclonal antibody against epidermal growth factor receptor (EGFR). Inhibiting EGFR prevents cell growth and induces apoptosis. It is used to treat solid cancers including head and neck cancer, non-small cell lung cancer, and colorectal cancer. Side effects include an acne like rash.  Similar to bevacizumab, ranibizumab is a monoclonal antibody w activity against VEGF; It is used for Tx of macular degeneration
  • 93. Marc Imhotep Cray MD Case#17 93 A 50-year-old woman returns to your oncology clinic for a follow-up visit. She is currently on a multidrug regimen for metastatic breast cancer. She appears to be responding well after the addition of a chemotherapeutic agent aimed at the HER-2/neu receptors within her cancer cells. While she is in the office, you schedule her for an imaging study to evaluate her heart. She asks why this test needs to be done and you remind her that, although the new medication is highly effective against her type of breast cancer, it can also cause cardiac dysfunction in up to 10% of patients. What is the Medication?
  • 94. Marc Imhotep Cray MD Trastuzumab 94 Mechanism of Action Trastuzumab acts as a monoclonal antibody against the HER-2/neu (erb-B2) receptor. The HER-2/neu receptor is involved in growth-promoting pathways of the cell; 20% to 30% of breast cancers will overexpress HER-2/neu, thereby contributing to uncontrolled cellular growth. Trastuzumab directly blocks HER-2/neu signaling and also induces antibody-dependent cell-mediated cytotoxicity. Clinical Uses Chemotherapeutic agent used to treat breast cancer and gastric cancer that overexpress HER-2 receptor. Side Effects/AEs Cardiac toxicity; neutropenia; GI upset.
  • 95. Marc Imhotep Cray MD Case#18 95 A 49-year-old man presents to your oncology office for follow-up of his chronic myelogenous leukemia, for which he is currently receiving chemotherapy. You ask him whether he is experiencing any nausea, vomiting, or diarrhea, all of which are common side effects of his medications. You also tell him that his blood counts are at the low end of normal, which is expected since his chemotherapeutic regimen can also cause bone marrow suppression. He was biochemistry major in college, so he is interested in knowing how the drugs that he is taking work. You explain to him that one of his medications acts by inhibiting ribonucleotide reductase, an enzyme involved in making deoxyribonucleotides, the building blocks for DNA. What is the Medication?
  • 96. Marc Imhotep Cray MD Hydroxyurea 96 Mechanism of Action Hydroxyurea inhibits ribonucleotide reductase, the enzyme responsible for reducing ribonucleotides to deoxyribonucleotides. Inhibition of this enzyme results in decreased DNA synthesis. Hydroxyurea has also been shown to increase levels of fetal hemoglobin (Hb F), although exact mechanism for this effect is unknown. Clinical Uses  Chemotherapeutic agent used to treat myeloproliferative disorders (e.g., CML, polycythemia vera) as well as melanoma  Hydroxyurea has also been used in the treatment of sickle cell disease to promote production of Hb F over hemoglobin S Side Effects/AEs Bone marrow suppression; GI upset. Note Hydroxyurea is a cell cycle-specific agent, acting during S phase of cell cycle
  • 97. Marc Imhotep Cray MD Case#19 97 A 64-year-old man with chronic myelogenous leukemia is being seen in your oncology clinic. He has been treated with α-interferon and cytarabine with no improvement. He asks you if there are any other options to treat his disease. You tell him that you would like to try a different agent that acts by inhibiting a tyrosine kinase on a mutated gene product, bcr-abl, which has been shown to be associated with his disease. What is the Medication?
  • 98. Marc Imhotep Cray MD Imatinib 98 Mechanism of Action Imatinib acts as a competitive inhibitor of tyrosine kinase enzymes in abl, c-kit, and PDGF-R (Platelet-derived growth factor receptors ), all genes involved in cellular growth. By inhibiting tyrosine kinase enzymes, activation of these genes is severely decreased, and cellular growth is slowed Clinical Uses Chemotherapeutic agent used to treat CML, GISTs (gastrointestinal stromal tumor), and brain tumors Side Effects/AEs Fluid retention (edema); nausea; rash; heart failure Important Note CML is commonly assoc. w translocations betw. chromosomes 9 and 22, yielding what is known as Philadelphia chromosome. The translocation produces a fusion protein, bcr-abl, which is constitutively active By inhibiting tyrosine kinase of abl portion of this mutated gene product wagents such as imatinib, cellular growth of CML is decreased
  • 99. Marc Imhotep Cray MD Types of Chemotherapeutic Agents: A Classification Summary 99
  • 100. Marc Imhotep Cray MD Alkylating Agents 100 Alkylating agents are most active in the resting phase of cell. These types of drugs are cell-cycle non-specific. Mustard gas derivatives: Mechlorethamine, Cyclophosphamide, Chlorambucil, Melphalan, and Ifosfamide.  Ethylenimines: Thiotepa and Hexamethylmelamine.  Alkylsulfonates: Busulfan.  Hydrazines and Triazines: Altretamine, Procarbazine, Dacarbazine and Temozolomide.  Nitrosureas: Carmustine, Lomustine and Streptozocin. Nitrosureas are unique because, unlike most types of chemo treatments, they can cross the blood-brain barrier. They can be useful in treating brain tumors.  Metal salts: Carboplatin, Cisplatin, and Oxaliplatin
  • 101. Marc Imhotep Cray MD Plant Alkaloids 101 Plant alkaloids are ChemoTx agents derived made from certain types of plants. Vinca alkaloids are made from periwinkle plant (catharanthus rosea). Taxanes are made from bark of Pacific Yew tree (taxus). Vinca alkaloids and taxanes are also known as antimicrotubule agents. The podophyllotoxins are derived from the May apple plant. Camptothecan analogs are derived from Asian "Happy Tree" (Camptotheca acuminata). Podophyllotoxins and camptothecan analogs are also known as topoisomerase inhibitors, which are used in certain types of chemotherapy. The plant alkaloids are cell-cycle specific. This means they attack cells during various phases of division  Vinca alkaloids: Vincristine, Vinblastine and Vinorelbine.  Taxanes: Paclitaxel and Docetaxel  Podophyllotoxins: Etoposide and Tenisopide  Camptothecan analogs: Irinotecan and Topotecan
  • 102. Marc Imhotep Cray MD Anti-tumor Antibiotics 102 Antitumor antibiotics are ChemoTx agents made from natural products produced by species of soil fungus Streptomyces. These drugs act during multiple phases of cell cycle and are considered cell-cycle specific. There are several types of antitumor antibiotics: •Anthracyclines: Doxorubicin, Daunorubicin, Epirubicin, Mitoxantrone, and Idarubicin •Chromomycins: Dactinomycin and Plicamycin •Miscellaneous: Mitomycin and Bleomycin
  • 103. Marc Imhotep Cray MD Antimetabolites 103 Antimetabolites are types of ChemoTx that are very similar to normal substances within cell. When cells incorporate these substances into cellular metabolism, they are unable to divide. Antimetabolites are cell- cycle specific. They attack cells at very specific phases in cycle. Antimetabolites are classified according to substances with which they interfere.  Folic acid antagonist: Methotrexate  Pyrimidine antagonist: 5-Fluorouracil, Foxuridine, Cytarabine, Capecitabine, and Gemcitabine  Purine antagonist: 6-Mercaptopurine and 6-Thioguanine  Adenosine deaminase inhibitor: Cladribine, Fludarabine, Nelarabine and Pentostatin
  • 104. Marc Imhotep Cray MD Topoisomerase Inhibitors 104 Toposiomerase inhibitors are types of ChemTx drugs that interfere with action of topoisomerase enzymes (topoisomerase I and II). During process of chemo Txs, topoisomerase enzymes control manipulation of structure of DNA necessary for replication. •Topoisomerase I inhibitors: Irinotecan, topotecan •Topoisomerase II inhibitors: Amsacrine, etoposide, etoposide phosphate, teniposide
  • 105. Marc Imhotep Cray MD Miscellaneous Antineoplastics 105 Several useful types of chemotherapy drugs are unique:  Ribonucleotide reductase inhibitor: Hydroxyurea  Adrenocortical steroid inhibitor: Mitotane  Enzymes: Asparaginase and Pegaspargase  Antimicrotubule agent: Estramustine  Retinoids: Bexarotene, Isotretinoin, Tretinoin (ATRA)  Beyond aforementioned types of ChemoTx agents, many other types of chemo treatments exist, such as targeted therapy, immunotherapy, and hormone therapy. Sources:  http://chemocare.com/chemotherapy/what-is-chemotherapy/types-of-chemotherapy.aspx  http://tmedweb.tulane.edu/pharmwiki/doku.php/cancer_chemotherapy
  • 106. 106 THE END See next slide for links to tools and resources for further study.
  • 107. Marc Imhotep Cray MD Further Study 107 Cloud Folder: CA ChemoTx Ppts, Notes, SDL Tutorials and Video